-->

Bank Soal UTBK (Ujian Tulis Berbasis Komputer) SNBPT Lengkap Dengan Jawaban dan Pembahasan

Bank Soal UTBK (Ujian Tulis Berbasis Komputer) SNBPT Lengkap Dengan Jawaban dan Pembahasan
Persiapan UTBK SNBPT



mediainggris.com - Ada kabar terbaru mengenai jenis soal dalam UTBK SNTB. Apakah sudah ada yang memberitahumu? Sepertinya sebagian besar dari kamu sudah mengetahui informasi ini. Bagaimana reaksimu? Terkejut? Senang? Emosional? Rasanya campur aduk, bukan?

Tepat sekali, dalam UTBK ini akan ada soal isian. Jadi, tidak hanya soal pilihan ganda, tetapi juga soal yang harus dijawab secara langsung di komputer. Tapi, jangan khawatir. Menurut SNPMB BPPP, soal-soal isian yang akan diuji tidak terlalu sulit.

Lalu, apa yang harus dilakukan setelah mengetahui ini? Jawabannya sederhana, belajar dan latihan. Latihan memahami soal, belajar logika dasar agar tidak mudah terjebak, dan masih banyak lagi.

Untuk membantu memahami jenis soal di UTBK SNTB, dalam artikel ini akan berikan kumpulan soal-soal UTBK dari berbagai sumber yang dapat kalian gunakan sebagai sarana latihan.

Berikut ini adalah Bank Soal UTBK SNBPT yang dilengkapi dengan jawaban dan pembahasannya.


Teks ini menjawab soal nomor 1-4. 
Indonesia memiliki kata yang cukup unik untuk mengekspresikan tertawa di internet. Netizen Indonesia sering kali menggunakan kata "wkwk" (baca: weka-weka) untuk ekspresi kala gembira, senang, dan geli tersebut. Penggunaan kata "wkwk" di Indonesia bisa dikatakan sangat unik. Warga dunia biasanya memilih istilah "laugh out loud" yang sering kali disingkat "LOL" atau "haha" untuk menggambarkan situasi tertawa. Adapun, "wkwk" hanya digunakan oleh orang Indonesia. Dari mana kata "wkwk" itu berasal? Para anggota Quora asal Indonesia pun berlomba-lomba menjawab pertanyaan tersebut. Salah satu pengguna Quora yang bernama Dio Wijayanto Nugroho menuturkan bahwa kata wkwk lahir di komunitas game online. (9) Penggunaan kata haha dianggap sangat formal di dunia game. (10) Pemain lebih memilih untuk menggunakan kata huehue dan 
huahua untuk ekspresi tertawa. (11) Nah, sama seperti gua menjadi gw, huruf U yang ada di huehue dan huahua diubah menjadi W sehingga pemain memilih menggunakan kata hwhwhw. (12) Kemudian, kata itu berubah lagi menjadi wkwk dengan alasan lebih mudah diketik dibandingkan haha. (13) Pemain game online bisa mengetikkan huruf W tanpa menggerakkan tangan kiri, sedangkan huruf K bisa diketik tanpa menggerakkan tangan kanan.

1. Kalimat yang bebas dari kesalahan penulisan adalah ....
A. (2)
B. (4)
C. (5)
D. (11)
E. (12)

Jawaban : D 
Pembahasan :
Soal ini sangat umum. Kesalahan yang dimaksud bisa disebabkan ketidakefektifan, ketidakbakuan, atau kesalahan dalam penggunaan ejaan. Kalimat yang bebas dari kesalahan penulisan adalah kalimat (11). Kalimat (2) kekurangan tanda koma di sebelum kata dan. Perincian yang lebih dari dua harus dipisahkan tanda koma pada setiap unsurnya. Kalimat (4) mengandung singkatan LOL yang menggunakan huruf miring. Singkatan tidak menggunakan huruf miring meskipun merupakan singkatan yang berasal dari bahasa asing. Kalimat (5) kelebihan tanda baca koma setelah kata adapun. Kalimat (12) mengandung kesalahan pembentukan kata. Kata diketik tidak tepat karena kata dasarnya adalah tik bukan ketik.

2. Konjungsi yang tepat untuk melengkapi kalimat (6) agar padu adalah
....
A. dengan demikian
B. lantas
C. jadi
D. sebenarnya
E. namun

Jawaban : B 
Pembahasan :
Soal ini menanyakan konjungsi antarkalimat yang tepat untuk mengisi bagian rumpang pada kalimat (6). Untuk menentukan konjungsi yang tepat, kita perlu mengetahui hubungan di antara kalimat (5) dan (6), (5) Adapun, wkwk hanya digunakan orang Indonesia. (6) [...], dari mana kata wkwk itu berasal?. Kalimat (5) membicarakan kata wkwk yang hanya digunakan di Indonesia. Kalimat (6) melanjutkan kalimat (5) dengan menanyakan dari mana asal kata wkwk tersebut. Konjungsi yang tepat untuk melanjutkan informasi adalah lantas yang bermakna lalu atau kemudian.
Pilihan jawaban A dan C tidak tepat karena konjungsi dengan demikian dan jadi digunakan untuk menunjukkan hubungan simpulan. Pilihan jawaban D tidak tepat karena konjungsi sebenarnya bukan merupakan konjungsi, melainkan adverbia. Pilihan jawaban E tidak tepat karena konjungsi namun digunakan untuk menunjukkan hubungan pertentangan.

3. Ide pokok paragraf kedua teks tersebut adalah ....
A. anggota Quora berlomba menjawab asal katawkwk
B. wkwk berasal dari komunitas game online
C. asal mula munculnya kata wkwk
D. wkwk muncul karena kebiasaan para pemain game online
E. salah satu pendapat mengenai asal kata wkwk

Jawaban : E 
Pembahasan :
Ide pokok adalah hal yang dibicarakan dalam sebuah teks atau paragraf. 
Ide pokok mencakup hal utama yang dibicarakan dan penjelasnya.

Paragraf kedua berisi penjelasan dari mana asal kata wkwk berasal. Penjelasan tersebut berasal dari seorang pengguna Quora yang bernama Dio Wijayanto. Artinya, asal kata wkwk dalam paragraf kedua merupakan sebuah pendapat yang dikemukakan oleh pengguna Quora tersebut.

Pilihan jawaban A tidak tepat karena paragraf kedua tidak menjelaskan perihal pengguna Quora yang berlomba menjawab pertanyaan mengenai asal kata wkwk.

Pilihan jawaban B tidak tepat karena kalimat ini berbunyi seakan-akan asal kata tersebut memang berasal dari sana, sedangkan pada awal kalimat dijelaskan bahwa itu hanya pendapat dari seorang pengguna Quora.

Pilihan jawaban C tidak tepat karena paragraf 2 membicarakan pendapat netizen terkait asal mula kata wkwk.

Pilihan jawaban D tidak tepat karena pernyataan tersebut merupakan penjelas bagi paragraf kedua.

4. Gagasan utama kalimat (8) adalah ...
A. Salah satu pengguna Quora bernama Dio Wijayanto.
B. Dio Wijayanto Nugroho menuturkanwkwk lahir di komunitas game 
online.
C. Pengguna Quora menuturkan wkwk lahir di komunitas game online.
D. Kata wkwk lahir di komunitas game online.
E. Nugroho menuturkan kata wkwk di komunitas game online.

Jawaban : D 
Pembahasan :
Gagasan utama kalimat adalah hal pokok yang dibicarakan dalam sebuah kalimat. Gagasan utama dalam sebuah kalimat biasanya terletak pada unsur intinya, yakni subjek dan predikat. Akan tetapi, pada sebuah kalimat aktif transitif tersebut terdapat kata bahwa sebagai kata penghubung untuk mendahului anak kalimat yang menjadi pokok kalimat. Oleh karena itu, gagasan utamanya terletak pada klausa objek tersebut.
Kalimat Salah satu pengguna Quora yang bernama Dio Wijayanto Nugroho menuturkan bahwa kata wkwk lahir di komunitas game online. mengandung objek yang diperluas. Dengan demikian, gagasan utama pada kalimat ini terletak pada klausa objeknya. Secara keseluruhan, struktur kalimat tersebut adalah 
Salah satu pengguna Quora yang bernama Dio Wijayanto Nugroho → subjek
menuturkan →  predikat
bahwa kata wkwk lahir di komunitas game online →  objek 
Analisis klausa anak perluasan objeknya adalah kata wkwk →  subjek 
lahir →  predikat
di komunitas game online →  keterangan.
Jadi, yang dibicarakan adalah kata wkwk lahir di komunitas game online. Keterangan pada kalimat tersebut berfungsi untuk memperjelas kalimatnya. Sementara itu, pilihan jawaban A, B, C, dan E tidak tepat karena subjek yang digunakan bukan kata wkwk.

5. Bacalah paragraf berikut!
(1) Para peserta lomba telah menyelesaikan karyanya. (2) Tibalah para juri untuk menilai karya-karya itu. (3) Dengan penuh antusias para peserta menunggu dan dengan perasaan berdebar-debar. (4) Mereka menanti keputusan juri. (5) Banyak karya-karya yang menjadi favorit dalam lomba ini, sehingga juri mengalami kesulitan menentukan pemenangnya.

Perbaikan kalimat tidak efektif pada nomor (5) adalah....

A. Banyak karya-karya favorit dalam lomba sehingga juri mengalami kesulitan menentukan
pemenangnya.
B. Banyak karya favorit lomba, sehingga juri mengalami kesulitan menentukannya.
C. Banyak karya-karya favorit lomba, sehingga juri mengalami kesulitan menentukannya.
D. Banyak karya yang menjadi favorit dalam lomba ini sehingga juri mengalami kesulitan menentukan pemenangnya.
E. Karya-karya favorit lomba ini banyak sehingga juri mengalami kesulitan menentukannya.

Jawaban: D
Pembahasan: Yang sesuai dengan syarat kehematan kalimat efektif dan yang lengkap adalah jawaban 'Banyak karya yang menjadi favorit dalam lomba ini sehingga juri mengalami kesulitan menentukan pemenangnya'.

6. Bukti kesuksesan artis yang menang di beberapa pemilihan kepada daerah (pilkada) atau "aman" menjadi anggota DPR mungkin menjadi daya tarik baru bagi artis lainnya saat ini. Baik artis maupun partai politik sama-sama terlihat saling "membutuhkan" untuk menaikkan popularitas. Hampir semua partai besar dipastikan mengusung nama-nama artis beken dalam daftar calon legislatifnya. Tak sebatas artis, tetapi juga MC, penyanyi, hingga pelawak. Akan tetapi, dalam sebuah survei terlihat bahwa masyarakat tidak terlalu berharap di masa depan para artis makin dominan dalam politik. Bahkan, sebagian besar cenderung tidak setuju jika di masa mendatang makin banyak artis yang menjadi anggota lembaga legislatif.

Makna kata survei dalam paragraf tersebut adalah...
A. Jajak pendapat
B. Sumbang saran
C. Kuesioner
D. Dengar pendapat
E. Penelitian

Jawaban: A
Pembahasan: Arti survei adalah jajak pendapat

7. Obat mujarab ini memiliki berbagai khasiat seperti melancarkan peredaran darah, [...] nafsu makan, dan meningkatkan stamina pria. Kata berimbuhan yang tepat untuk mengisi kekosongan pada kalimat tersebut adalah....
A. menambahkan
B. menambahi
C. menambah
D. ditambahkan
E. ditambahi

Jawaban: A
Pembahasan: Jawaban yang tepat adalah menambahkan

8. Cermati paragraf berikut!
....Bintang-bintang di langit bertaburan dengan memancarkan cahaya yang berkelip-kelip. Langit tidak. tertutup awan. Desir angin yang bertiup terasa lembut menyentuh kulit. Kalimat yang tepat untuk melengkapi paragraf tersebut adalah
A. Suasana cukup mengesankan
B. Udara mala mini lembut sekali
C. Malam ini angin bertiup sepoi-sepoi basah
D. Malam ini indah dan udara cerah sekali.
E. Suasana malam yang mengharukan

Jawaban: D
Pembahasan: Penggambaran dari kalimat tersebut adalah penggambaran yang indah dan cerah maka jawaban yang tepat adalah Malam ini indah dan udara cerah sekali.

9. Bacalah kalimat berikut dengan saksama!
Bambu adalah material alami yang sering dipandang sebelah mata ...ia memiliki banyak kelebihan, murah, tahan air, ... mempunyai banyak manfaat.

Kata penghubung yang tepat untuk melengkapi kalimat tersebut 
adalah....
A. karena, serta, dan
B. sehingga, antara, serta
C. namun, juga, dan
D. padahal, yakni, dan
E. bahkan, missal, juga

Jawaban: D
Pembahasan: Jawaban yang tepat adalah padahal, yakni, dan

10. Bacalah teks bacaan berikut dengan saksama!
[...] tahun 1937, penemuan alat-alat serpih di Bukit Ngebung oleh von Koenigswald menjadi cikal bakal ketenaran Sangiran di dunia ilmu pengetahuan khususnya dalam bidang Paleantropologi. Penemuan tersebut [...] munculnya temuan-temuan baru lainnya, salah satunya adalah fosil manusia purba. Hal tersebut manusia purba purba bahwa Sangiran sebagai situs manusia purba yang memiliki kekayaan tinggalan manusia beserta budaya. (Menyadur dari artikel M. Rais Fathoni yang berjudul, "Analisis Morfologi Berangkal Andesit Yang Mengalami Pelapukan Membola Hubungannya Dengan Morfologi Bola Batu Di Sangiran").

Kata berimbuhan yang tepat untuk mengisi titik pada kalimat kedua 
adalah...
A. menginspirasi
B. memotivasi
C. menjadikan
D. memperoleh
E. menyatakan

Jawaban: A
Pembahasan: Jawaban yang tepat adalah kata menginspirasi

Bacalah teks di bawah ini dengan seksama untuk menjawab pertanyaan nomor 11!
Dalam kamus The Webster's New Collegiate mendefinisikan prokrastinasi sebagai pengunduran terhadap suatu hal yang disengaja dan biasanya disertai dengan suasana perasaan tidak suka mengerjakan sesuatu yang harus dikerjakan sekarang.
Prokrastinasi juga bisa disebutkan sebagai sebuah kecenderungan  menunda-nunda penyelesaian suatu tugas atau pekerjaan (Fauziah, 2015: 124). Penyebutan untuk orang yang suka menunda-nunda tugas atau pekerjaan disebut prokrastinator.
Solomon dan Rothblum (1984) menyatakan bahwa prokrastinasi  merupakan kecenderungan menunda tugas dengan melakukan aktivitas lain yang tidak berguna (...) tugas menjadi terhambat dan tidak selesai tepat waktu. Sama seperti pendapat Steel (2010), prokrastinasi adalah sebuah perilaku menunda dengan sengaja kegiatan yang diinginkan atau dibutuhkan walaupun individu mengetahui bahwa perilaku penundaannya berdampak buruk ke depannya (dalam Fauziah, 2015: 125).

11. Konjungsi yang tepat untuk mengisi bagian rumpang pada paragraf 
kedua adalah....
A. meskipun
B. walaupun
C. sehingga
D. karenanya
E. sementara itu

Jawaban: C
Pembahasan: Konjungsi yang tepat adalah sehingga

Bacalah teks berikut untuk menjawab pertanyaan nomor 12-16! 
[...](1) Selama kurun waktu 10 tahun terakhir, kondisi inflasi Indonesia belum stabil. (2) Kehidupan [...] tidak lepas dari inflasi yang terlihat dari naiknya harga barang-barang kebutuhan pokok dari tahun ke tahun. (3) Definisi konsep nilai waktu uang adalah jumlah [...] yang sama hari ini lebih bermakna daripada esok. (4) Artinya, inflasi dapat mengurangi daya beli uang yang kita miliki sekarang karena menumbuhkan harga terus-menerus. (5) Akibatnya, pendapatan [...] menurun sehingga banyak masyarakat semakin susah memenuhi kebutuhan sehari-harinya. (6) Oleh karena itu, investasi diperlukan untuk melindungi nilai aset atau [...] sehingga dapat digunakan sebagai persiapan kebutuhan di masa depan.

12. Judul yang paling tepat untuk melengkapi tulisan di atas adalah....
A. Daya Beli Uang
B. Konsep Nilai Waktu Uang
C. Inflasi Menggerogoti Nilai Uang
D. Kehidupan Perekonomian Indonesia
E. Inflasi Meningkatkan Daya Beli Uang

Jawaban: A
Pembahasan: Definisi dari Judul adalah nama yang dipakai untuk buku atau bab dalam buku yang dapat menyiratkan secara pendek isi atau maksud buku atau bab itu. Judul yang tepat yang menyiratkan isi teks tersebut yang tepat adalah "Daya Beli Uang".

13. Kata yang paling tepat untuk melengkapi titik-titik pada kalimat (2) adalah ...
A. berekonomi
B. keekonomian
C. perekonomian
D. pengekonomian
E. mengekonomikan

Jawaban: C
Pembahasan: Jawaban yang tepat adalah kata perekonomian

14. Kata yang paling tepat untuk melengkapi titik-titik pada kalimat (3) adalah ....
A. kas
B. prive
C. utang
D. modal
E. Piutang

Jawaban: A
Pembahasan: Jawaban yang tepat adalah kata kas

15. Kata yang paling tepat untuk melengkapi titik-titik pada kalimat (5) adalah....
A. real
B. riil
C. mikro
D. makro
E. hari-hari

Jawaban: B
Pembahasan: Jawaban yang tepat dan berkaitan adalah kata riil

16. Kata yang paling tepat untuk melengkapi titik-titik pada kalimat (6) 
adalah....
A. dana
B. entiti
C. derma
D. entitas
E. Ekuitas

Jawaban: A
Pembahasan: Dana merupakan uang yang disediakan untuk suatu keperluan atau biaya. Investasi berkaitan dengan melindungi dana dan aset karena merupakan bentuk penanaman uang atau modal untuk memperoleh keuntungan. Jadi, kata yang paling tepat untuk melengkapi titik-titik pada kalimat 6 adalah dana. Kata-kata lainnya tidak tepat karena entitas berarti satuan yang berwujud, ekuitas berarti kepemilikan nilai uang, dan derma berarti bantuan uang.

17. Persebaran virus corona (Covid -19) semakin meluas, mudahnya penyebaran virus sedikit banyak dipengaruhi oleh (14) kuantitas kebiasaan hidup masyarakat Indonesia. Mulai dari mencuci tangan, menggunakan pembersih tangan hingga mengonsumsi vitamin tambahan 
perlu dilakukan secara (15) serentak, sebagai upaya pencegahan penyebaran virus tersebut. 
Pemberitaan yang semakin (16) gencar tentang bahaya virus juga semakin marak, jangan lupa juga dari (17) hoax bertebaran di dunia  maya. Sebagai masyarakat yang cerdas, kita sulit untuk selalu berpikir kritis, memilih dan memilah berita mana (18) yang relevan dipilih. Rata- 
rata memang konten dari isi berita di era sekarang tentang hal-hal yang membuat masyarakat khawatir dan cemas. Oleh karenanya,
A. TIDAK PERLU DIPERBAIKI
B. "Kuantitas"
C. Kuantitas
D. Kualitas
E. "Kualitas"

Jawaban: D
Pembahasan: Karena yang berkaitan dengan kebiasaan hidup adalah kualitas bukan kuantitas

18. Proses membaca yang dimulai dari memahami bacaan secara literal kemudian menginterpretasikan dan memberikan reaksinya berupa penilaian terhadap apa yang dikatakan penulis, dilanjutkan dengan mengembangkan pemikiran-pemikiran sendiri untuk membentuk gagasan, wawasan, pendekatan, dan pola-pola pikiran baru. Membaca dengan cara tersebut merupakan membaca dengan pemahaman...
A. literal
B. kritis
C. kreatif
D. interpretatif
E. Aspiratif

Jawaban: C
Pembahasan: Jawaban yang tepat adalah pemahaman kreatif yaitu proses membaca kreatif menurut Syafi'l (1999:36).

(1)Kasus korupsi masih mengisi jagat pemberitaan di Indonesia.
(2)Dalam beberapa bulan terakhir, sejumlah pejabat pemerintah tersandung dugaan kasus korupsi. (3)Misalnya, bekas Menteri Sosial Juliari Batubara, bekas Menteri Kelautan dan Perikanan Edhy Prabowo, dan Gubernur nonaktif Sulawesi Selatan Nurdin Abdullah. (4)Ironisnya, sebagian kasus korupsi yang kini tengah diproses Komisi Pemberantasan Korupsi (KPK) berhubungan dengan bantuan sosial dan sejumlah program terkait penanganan pandemi Covid-19. (5)[...] kasus korupsi dana bantuan sosial yang menjerat Juliari, terdapat pula dugaan korupsi pengadaan barang terkait tanggap darurat bencana Covid-19 di Dinas Sosial Kabupaten Bandung Barat, Jawa Barat. (6)Terbukti sudah bahwa pejabat kita tak memikirkan rakyat.

19. Kalimat tidak efektif pada paragraf di atas adalah ...
A. Kalimat 1
B. Kalimat 2
C. Kalimat 3
D. Kalimat 4
E. Kalimat 5

Jawaban: C
Kalimat tidak efektif pada paragraf tersebut adalah kalimat 3 > kalimat 
tersebut tidak terdapat subjek dan predikat hanya berisi keterangan saja.

20. Kalimat yang tidak koheren pada teks di atas terdapat pada kalimat ...
A. (2)
B. (3)
C. (4)
D. (5)
E. (6)

Jawaban: E
Kalimat yang tidak koheren atau kalimat yang tidak mendukung gagasan pokok adalah kalimat 6 > Terbukti sudah bahwa pejabat kita tak memikirkan rakyat. > kalimat tersebut tidak sesuai dengan gagasan pokoknya yang lebih menekankan korupsi.

Perhatikan teks berikut!
Persoalan korupsi ibarat fenomena gunung es. Hasil jajak pendapat Litbang 58 Kompas pada 16-18 Maret 2021 menunjukkan, persoalan korupsi jamak ditemukan 59 responden yang tersebar di 34 provinsi di Indonesia. Sebanyak 70,6 persen responden yang menyatakan pernah menemui kasus korupsi di lingkungan sekitar dalam kurun waktu satu tahun terakhir.

21. ....
A. TIDAK PERLU DIPERBAIKI
B. KOMPAS
C. Kompas
D. "Kompas"
E. Kompas

Jawaban: C
Penulisan Kompas seharusnya dicetak miring

22. ....
A. TIDAK PERLU DIPERBAIKI
B. Respondent
C. Responsif
D. Responden
E. Respon

Jawaban: A
Penulisan kata responden sudah benar sesuai KBBI 

23. ...
A. TIDAK PERLU DIPERBAIKI
B. Sebanyak 70,6 persen responden yang menyatakan pernah menemukan kasus korupsi di lingkungan sekitar dalam kurun waktu satu tahun terakhir
C. Sebanyak 70,6 persen responden menyatakan pernah menemui kasus korupsi di lingkungan sekitar dalam kurun waktu satu tahun terakhir
D. Sebanyak 70,6 persen responden menyatakan pernah menemukan kasus korupsi di lingkungan sekitarnya dalam kurun waktu satu tahun terakhir
E. Sebanyak 70,6 persen responden yang menyatakan pernah menemui kasus korupsi di lingkungan sekitar dalam kurun waktu satu tahun terakhir

Jawaban: D
Kalimat tersebut diperbaiki dengan cara > 1) menghilangkan kata yang, 2) mengganti kata menemui dengan menemukan,3) kata sekitar menjadi sekitarnya.

Perhatikan teks berikut!
Melihat kucing tiba-tiba mengonsumsi rumput menimbulkan rasa kekhawatiran. Tak jarang juga para pemilik kucing mencoba mencegah kelakuan kucingnya yang tiba-tiba mengunyah rumput liar di sekitaran rumah. Sebuah penelitian mengatakan bahwa hal ini dilakukan karena tanaman yang tidak dapat dicerna kucing itu justru dapat membersihkan pencernaannya dari cacing parasit dengan memuntahkannya.

24. Manakah pernyataan berikut yang akan MEMPERKUAT hasil penelitian tersebut?
A. Rumput menjadi makanan sehat bagi kucing karena mudah dicerna.
B. Kucing yang muntah setelah memakan rumput memiliki usia yang lebih panjang.
C. Kucing yang memakan rumput memiliki cacing parasit dalam organ pencernaannya.
D. Pemilik kucing rajin memberikan rumput untuk membersihkan cacing parasit pada kucing.
E. Pemilik kucing mencegah kucingnya untuk memakan rumput karena rumput tidak dapat dicerna kucing.

Jawaban B
Pembahasan
Pernyataan yang memperkuat adalah pernyataan yang bersifat mendukung. Hasil penelitian mengatakan bahwa kucing memakan rumput karena tanaman yang tidak dapat dicerna kucing itu justru dapat membersihkan pencernaannya dari cacing parasit dengan memuntahkannya. Berikut ini adalah hasil analisis dari setiap pilihan jawaban terhadap hasil penelitian.
Pilihan jawaban A tidak tepat karena tidak sesuai dengan hasil penelitian. Rumput dapat dikatakan menjadi obat bagi kucing untuk membersihkan pencernaannya karena rumput sulit dicerna sehingga pernyataan pada pilihan jawaban A tidak memperkuat hasil penelitian.
Pilihan jawaban B tepat karena menurut hasil penelitian, rumput yang dimakan kucing akan membuat kucing muntah sehingga dapat membersihkan pencernaannya. Dengan bersihnya pencernaan dan terbebas dari cacing parasit, kucing akan memiliki usia hidup yang lebih panjang. Oleh karena itu, pernyataan pada pilihan jawaban B memperkuat hasil penelitian.
Pilihan jawaban C tidak tepat karena tidak memperkuat hasil penelitian. Hasil penelitian menjelaskan tentang manfaat rumput bagi kebersihan pencernaan kucing, sedangkan pilihan jawaban C membahas tentang kucing yang memakan rumput memiliki cacing parasit dalam organ pencernaannya.
Pilihan jawaban D tidak tepat karena tidak sesuai dengan hasil penelitian. Dalam hasil penelitian tidak dijelaskan mengenai pemilik kucing yang memberikan rumput untuk makan kucing. Dalam teks juga dikatakan bahwa tak jarang pemilik kucing mencoba mencegah kelakuan kucingnya yang tiba-tiba mengunyah rumput liar di sekitaran rumah. Artinya, kucing tidak diberikan makan rumput oleh pemiliknya, melainkan memakannya sendiri.
Pilihan jawaban E tidak tepat karena tidak memperkuat hasil penelitian. Hasil penelitian menjelaskan manfaat memakan rumput bagi kucing, sedangkan pernyataan pada pilihan jawaban E menjelaskan keadaan yang dikatakan pada kalimat kedua dalam teks.

Perhatikan wacana berikut!
PT Eka Sari Lorena Transport Tbk mencatatkan bahwa keseluruhan nilai beban perusahaan dari layanan bus antarkota antarprovinsi (AKAP) itu lebih tinggi ketimbang pendapatan pada masa pandemi. PT Weha Transportasi Indonesia Tbk, perusahaan layanan bus pariwisata, mengemukakan terjadinya perubahan pendapatan dari Rp71,9 miliar menjadi Rp70,5 miliar. Perusahaan taksi PT Blue Bird Tbk, pendapatannya rontok 49,4 persen menjadi Rp2,1 triliun pada periode yang sama. Perusahaan taksi lain, PT Express Transindo Utama, berhasil menurunkan nilai kerugian pada 2020 lalu menjadi Rp53,2 miliar dari sebelumnya Rp276,1 miliar.

25. Pernyataan berikut ini yang tepat untuk dijadikan KESIMPULAN dari 
wacana tersebut adalah ...
A. Transportasi publik menjadi salah satu sektor usaha yang ikut terdampak pandemi Covid-19.
B. Pendapatan perusahaan bus pariwisata pada tahun pandemi menurun lebih dari 50 persen.
C. Pendapatan perusahaan taksi tidak sebanding dengan beban perusahaan sehingga mengalami kerugian.
D. Selama masa Pembatasan Sosial Berskala Besar (PSBB), semua jenis transportasi darat mengalami kerugian.
E. Penurunan kinerja semua perusahaan transportasi antarprovinsi disebabkan karena pelaksanaan pembatasan sosial.

Untuk mengetahui kesimpulan yang sesuai dengan wacana, dapat diperiksa masing-masing pernyataan pada pilihan jawaban.
Pilihan jawaban A tepat. Perhatikan bahwa empat perusahaan yang disebutkan pada wacana tersebut merupakan perusahaan yang bergerak di bidang jasa transportasi publik. Kemudian, diceritakan bahwa masing-masing perusahaan mengalami kerugian yang ditandai dengan frasa "... beban perusahaan ... lebih tinggi ketimbang pendapatan", "... perubahan pendapatan dari Rp71,9 miliar menjadi Rp70,5 miliar (terjadi penurunan).", "... pendapatannya rontok ...", dan "... berhasil menurunkan nilai kerugian ... (masih merugi)". Selain itu, semua itu terjadi karena pandemi Covid-19 yang melanda negeri.
Pilihan jawaban B tidak tepat karena dikatakan perusahaan bus pariwisata, padahal ada perusahaan taksi juga yang pendapatannya juga menurun akibat pandemi. Frasa perusahaan bus pariwisata tidak meliputi frasa transportasi publik pada umumnya.
Pilihan jawaban C tidak tepat karena dikatakan perusahaan taksi, padahal ada perusahaan bus juga yang mengalami kerugian akibat pandemi. Frasa perusahaan taksi tidak meliputi frasa transportasi publik pada umumnya.
Pilihan jawaban D tidak tepat karena dikatakan semua jenis transportasi darat, padahal ada kereta api yang juga merupakan transportasi darat, tetapi tidak diceritakan apakah mengalami kerugian atau tidak.
Pilihan jawaban E tidak tepat karena dikatakan penurunan kinerja semua perusahaan transportasi antarprovinsi, padahal bisa jadi terdapat perusahaan transportasi antarprovinsi yang tidak mengalami penurunan kinerja pada masa pelaksanaan pembatasan sosial.

26. Lima sekawan Sano, Joko, Adi, Rimba, dan Ratu selalu semangat berangkat bersama menuju sekolah. Joko selalu menjemput Sano, setelah ia dijemput oleh Adi. Rimba menjadi anak terakhir yang dijemput. Sementara rumah Ratu terletak di antara rumah Joko dan rumah Adi. 

Berikut ini pernyataan yang BENAR adalah ...
A. Rumah Ratu terletak paling jauh
B. Rumah Adi terletak paling jauh
C. Rumah Rimba terletak paling jauh
D. Rumah Sano terletak paling dekat
E. Rumah Adi terletak paling dekat

Jawaban: B 
Pembahasan:
Logikanya, siapa yang berangkat lebih dulu berarti rumahnya lebih jauh ke sekolah. Dari soal potensi kognitif di atas dapat kita ketahui bahwa rumah Adi lebih jauh dari rumah Joko. Selanjutnya, Sano rumahnya lebih dekat dari rumah Joko. Sementara itu, rumah Ratu berada di antara rumah Joko dan Adi. Sedangkan, rumah Rimba adalah yang paling dekat karena dia dijemput paling terakhir.
Apabila dituliskan berdasarkan urutan dari yang terdekat hingga terjauh adalah sebagai berikut:
Rimba - Sano - Joko - Ratu - Adi

27. Berdasarkan data nilai matematika siswa di suatu sekolah, diketahui nilai Ahmad lebih tinggi dari nilai Bima. Nilai Dimas tidak lebih tinggi dari nilai Fina, tetapi lebih tinggi daripada nilai Erin. Nilai Bima sedikit lebih tinggi dari nilai Cici. Jika nilai Cici lebih tinggi dari nilai Fina, maka pernyataan berikut yang pasti benar adalah ...
A. Nilai kedua terendah didapat oleh Erin.
B. Nilai Cici lebih rendah dari nilai Dimas.
C. Nilai Dimas berada di urutan ketiga tertinggi.
D. Fina memiliki nilai yang lebih tinggi daripada Ahmad.
E. Dimas memiliki nilai yang lebih rendah dari nilai Bima.

Jawaban : E 
Pembahasan :
Untuk menentukan pernyataan yang pasti benar berdasarkan teks tersebut, dibutuhkan urutan lengkap dari nilai-nilai tersebut.
Berdasarkan informasi pada soal, diketahui bahwa:
nilai Ahmad lebih tinggi dari nilai Bima,
nilai Bima sedikit lebih tinggi daripada nilai Cici,
nilai Cici lebih tinggi dari nilai Fina,
nilai Dimas tidak lebih tinggi dari Fina, dan
nilai Dimas lebih tinggi daripada nilai Erin.
Oleh karena itu, diperoleh urutan nilai dari yang tertinggi ke terendah sebagai berikut.
Ahmad - Bima - Cici - Fina - Dimas - Erin
Dengan demikian, pernyataan yang benar adalah Dimas memiliki nilai yang lebih rendah dari nilai Bima.
Alasan pilihan jawaban lain tidak tepat adalah sebagai berikut. Pilihan jawaban A tidak tepat, karena nilai kedua terendah didapatkan oleh Bima, bukan Erin.
Pilihan jawaban B tidak tepat, karena nilai Ciri lebih tinggi dari nilai Dimas.
Pilihan jawaban C tidak tepat, karena Dimas berada diurutan kelima. 
Pilihan jawaban D tidak tepat, karena Fina memiliki nilai lebih rendah dari Ahmad.
Jadi, jawaban yang tepat adalah E.

Teks
Pemerintah berencana mengembalikan program Kartu Prakerja sesuai fungsi semula, yakni meningkatkan kapasitas pekerja. Pemerintah diingatkan agar rencana mengubah program menjadi bukan lagi semibantuan sosial itu tidak dilakukan terburu-buru. Alasannya, pekerja yang mengalami dampak pandemi Covid-19 masih membutuhkan bantalan sosial hingga situasi berangsur-angsur normal. Deputi Bidang Ekonomi Digital, Ketenagakerjaan, dan UMKM Kementerian Koordinator Bidang Perekonomian Rudy.

Salahuddin menjelaskan, perubahan skema program Kartu Prakerja kemungkinan besar dimulai pada semester II-2021. Program Kartu Prakerja memasuki gelombang terakhir pada semester I-2021, dengan penyediaan kuota peserta 2,7 juta orang.

Selama pandemi, Kartu Prakerja menjadi program semi-bansos berupa pemberian bantuan biaya kelas pelatihan dalam jaringan Rp 1 juta dan insentif Rp 600.000 per bulan selama empat bulan. Porsi biaya untuk insentif bansos lebih besar dibandingkan dengan insentif pelatihan. "Semula, kami meminta semester I ini tetap berupa semi-bansos karena kita masih perlu mendorong konsumsi masyarakat. Akan tetapi, pada semester II, kalau vaksinasi sudah lancar, kita sudah bisa mulai melakukan uji coba," kata Rudy saat dihubungi di Jakarta, Minggu (28/3/2021).

la menambahkan, hasil penyelenggaraan program Kartu Prakerja pada semester I-2021 akan dievaluasi untuk menentukan kebijakan pada semester II-2021. Uji coba pada semester II melalui pelatihan luar jaringan di sejumlah daerah zona hijau yang diberi izin oleh Satuan Tugas Penanganan Covid-19. Menurut dia, pengembalian Kartu Prakerja ke fungsi awal sebagai peningkatan kapasitas pekerja akan membawa konsekuensi bobot program lebih berat pada kelas-kelas pelatihan, bukan lagi insentif bansos bagi pekerja. "Begitu kita buka pelatihan luring, kita harus ubah kondisinya untuk peningkatan kompetensi, bukan lagi instrumen bansos," kata Rudy. Namun, rencana ini belum diputuskan karena harus disepakati rapat Komite Cipta Kerja yang dikoordinasikan Kementerian Koordinator Bidang Perekonomian.

Direktur Eksekutif Manajemen Pelaksana Kartu Prakerja, Denni Puspa Purbasari mengatakan tujuan awal Kartu Prakerja adalah meningkatkan kompetensi, produktivitas, daya saing, dan kewirausaaan angkatan kerja. Apalagi akibat pandemi, digitalisasi adan otomatosasi terjadi lebih cepat dan mengubah pasar kerja. "Maka narasinya keliru kalau mengatakan yang berhak mendapatkan Kartu Prakerja adalah korban pemutusan hubungan kerja dan para penganggur. Sejak awal kami Menyusun aturannya untuk semua tingkatan kerja," kata Deni.

28. Berdasarkan paragraf 1, pernyataan yang tidak benar adalah ....
A. Pemerintah berencana mengembalikan program Kartu Prakerja sesuai fungsinya.
B. Pemerintah diingatkan agar rencana mengubah program tidak dilakukan terburu-buru.
C. Fungsi Kartu Prakerja adalah meningkatkan kapasitas pekerja.
D. Pekerja yang terdampak pandemi Covid-19 masih membutuhkan bantuan
E. Pemerintah merencanakan pengubahan program menjadi semibantuan sosial.

Jawaban: E
Pernyataan yang benar sesuai teks adalah > Pemerintah berencana mengembalikan program Kartu Prakerja sesuai fungsi semula, yaitu untuk meningkatkan kapasitas pekerja. Jadi pernyataan > Pemerintah merencakan pengubahan program menjadi semibantuan sosial > pernyataan tidak benar.

29. Berdasarkan paragraf 1, simpulan yang benar adalah...
A. Pemerintah berencana mengembalikan program Kartu Prakerja sesuai fungsi semula, yaitu bertujuan untuk meningkatkan kapasitas pekerja.
B. Skema perubahan Kartu Prakerja kemungkinan besar akan dimulai pada semester II-2021.
C. Program Kartu Prakerja pada semester I-2021 telah menyediakan kuota peserta 2,7 juta orang.
D. Pekerja yang terdampak pandemi Covid-19 masih membutuhkan bantalan sosial.
E. Pemerintah tidak akan terburu-buru mengubah program Kartu Prakerja bukan lagi semibantuan sosial. 

Jawaban: A
Pembahasan: Simpulan yang benar sesuai teks di paragraf 1 adalah > Pemerintah berencana mengembalikan program Kartu Prakerja sesuai fungsi semula, yaitu bertujuan untuk meningkatkan kapasitas pekerja.

30. Berdasarkan paragraf 2, Jika vaksinasi sudah lancar, simpulan YANG PALING MUNGKIN BENAR mengenai program Kartu Prakerja adalah...
A. Pemberian bantuan biaya kelas pelatihan dalam jaringan akan ditingkatkan.
B. Porsi biaya untuk insentif bansos lebih besar dibandingkan dengan insentif pelatihan.
C. Perubahan skema program Kartu Prakerja akan dilakukan uji coba pada semester II-2021.
D. Peningkatan untuk insentif bansos.
E. Pada semester I-2021 pemerintah tetap akan memberikan bantuan.

Jawaban: C
Pembahasan: Jika vaksinasi sudah lancar, simpulan yang paling mungkin benar sesuai isi paragraf 2 adalah > Perubahan skema program Kartu Prakerja akan dilakukan uji coba pada semester II-2021. Hal tersebut dilakukan untuk mengembalikan fungsi Kartu Prakerja untuk meningkatkan kapasitas pekerja melalui program pelatihan.

31. Berdasarkan paragraf 3, simpulan yang benar mengenai pengembalian fungsi Kartu Prakerja adalah...
A. Hasil penyelenggaraan Kartu Prakerja semester I-2021 akan dievaluasi.
B. Konsekuensi bobot program lebih berat pada kelas pelatihan, bukan lagi insentif bansos.
C. Peningkatan kompetensi sejalan dengan pemberian bansos.
D. Pelatihan luring akan diujicobakan pada semester II-2021.
E. Rencana pengubahan program akan dilaksanakan jika telah ada kesepakatan Komite Cipta Kerja.

Jawaban: B
Pembahasan: Sesuai isi paragraf 3 simpulan yang benar mengenai pengembangan fungsi Kartu Prakerja adalah > Konsekuensi bobot program lebih berat pada kelas pelatihan, bukan lagi insentif bansos. Hal tersebut sesuai dengan tujuan program Kartu Prakerja.

32. Berdasarkan paragraf 3, pernyataan yang tidak benar adalah...
A. Uji coba pelatihan luar jaringan dilakukan di daerah zona hijau.
B. Hasil penyelenggaraan program Kartu Prakerja pada semester II-2021 akan dievaluasi.
C. Pelatihan luring dititikberatkan pada peningkatan kompetensi.
D. Rencana pengubahan skema program belum akan dilaksanakan sebelum disepakati oleh Komite Cipta Kerja.
E. Komite Cipta Kerja dikoordinasikan Kementerian Koordinator Bidang Perekonomian.

Jawaban: B
Pembahasan: Pernyataan yang tidak benar adalah > Hasil penyelenggaraan program Kartu Prakerja pada semester II- 2021 akan dievaluasi. Hal tersebut tidak sesuai dengan isi teks seharusnya yang akan dievaluasi program Kartu Prakerja semester I-2021.

33. Kendaraan membutuhkan bahan bakar. 
Motor adalah salah satu kendaraan.
A. Motor membutuhkan bahan bakar.
B. Motor tidak membutuhkan bahan bakar.
C. Bahan bakar yang diperlukan untuk motor hanya sedikit.
D. Motor digunakan tidak membutuhkan bahan bakar.
E. Bahan bakar tidak untuk kendaraan motor.

Jawaban: A
Pembahasan: Karena semua kendaraan butuh bahan bakar, motor pun membutuhkan bahan bakar.

34. Semua sinden pandai menari tradisional. Sinden yang bernama Wening tidak dapat menari tradisional di panggung.
A. Wening bukan sinden panggung.
B. Wening adalah sinden sampingan.
C. Wening adalah sinden amatir.
D. Wening tidak pandai menari.
E. Wening bukan sinden, tetapi penari.

Jawaban: D
Pembahasan: Semua sinden pandai menari tradisional. Sinden yang bernama Wening tidak dapat menari tradisional di panggung. Jadi, Wening tidak pandai menari.

35. Semua kue adalah manis 
Semua kue adalah coklat
A. Semua coklat adalah manis
B. Sebagian coklat tidaklah manis
C. Sebagian kue manis bukan coklat
D. Coklat bukanlah kue manis
E. Semua kue coklat tidak manis

Jawaban: A
Pembahasan: Oleh karena kedua pernyataan bersifat semua, jadi semua coklat adalah manis.

Perhatikan teks ini!
Pada sebuah pertemuan yang terdiri atas para koki professional, yaitu Chef Randy, Chef Rani, Chef Agil, dan Chef Vero. Salah seorang dari mereka merupakan ahli dalam masakan Indonesia, sementara yang lainnya ahli dalam masakan Eropa, Italia, dan Cina. Pada suatu waktu, mereka dipertemukan dalam suatu acara memasak sebagai juri dan duduk di meja persegi dengan susunan sebagai berikut.
- Ahli masakan Indonesia duduk di sebelah kiri Chef Randy. 
- Ahli masakan Cina duduk bersebrangan dengan Chef Vero. 
- Chef Rani dan Chef Agil duduk berdekatan.
- Sementara itu, salah seorang Chef duduk di sebelah kiri ahli masakan  Eropa.
- Ahli masakan Italia bersebrangan dengan Chef Agil

36. Siapakah yang ahli dalam masakan Italia?
A. Chef Randy
B. Chef Rani
C. Chef Agil
D. Chef Vero
E. Semua Chef

Jawaban: A 
Pembahasan:
15/40
- Pilihlah satu posisi sebagai patokan, missal Chef Randy di nomor 1, 
sehingga nomor 4 pasti ahli masakan Indonesia. (syarat 1)
- Chef Vero tidak bisa di nomor 3 karena Chef Rani dan Chef Agil tidak berdekatan. (syarat 3)
- Chef Vero tidak bisa di nomor 2 karena di seberang nomor 2 bukan ahli masakan Cina. (syarat 2)
- No. 2 seorang ahli masakan Cina dan diisi oleh Chef Rani, sehingga Chef Agil di nomor 3 dan sebagai ahli masakan Eropa. (syarat 4)

37. Yang ahli dalam masakan Cina dan Eropa adalah ....
A. Chef Randy dan Chef Rani
B. Chef Rani dan Chef Agil
C. Chef Agil dan Chef Vero
D. Chef Randy dan Chef Vero
E. Chef Vero dan Chef Rani

Jawaban: B
Pembahasan: Yang ahli dalam masakan Cina adalah Rani, sementara Agil ada di nomor 3 yang merupakanahli masakan Eropa.

38. Urutan Negara yang sesuai dengan keahlian dari Chef Randy, Chef Rani, Chef Agil, dan Chef Vero berikut yang benar adalah....
A. Indonesia, Italia, Eropa, dan Cina
B. Cina, Italia, Eropa, dan Indonesia
C. Eropa, Cina, Indonesia, dan Italia
D. Italia, Cina, Eropa, dan Indonesia
E. Indonesia, Cina, Eropa, dan Italia

Jawaban: D 
Pembahasan:
Chef Randy : Italia
Chef Rani : Cina
Chef Agil : Eropa
Chef Vero : Indonesia

Bacalah Teks dibawah ini!
Boyolali - sebanyak empat kecamatan di Boyolali belum tersentuh 
program insentif pajak bumi dan bangunan (PBB) meskipun termasuk lahan pertanian pangan berkelanjutan (LP2B). Empat kecamatan itu meliputi Kecamatan Juwangi, Kecamatan Selo, Kecamatan Ampel, dan Kecamatan Mojosongo. Keempat kecamatan tersebut belum tersentuh program insentif pajak karena sedikitnya jumlah sawah yang bisa digunakan untuk menanam padi dan menggunakan irigasi teknis. Sementara program LP2B dilakukan di lahan sawah beririgasi teknis.

Sebagai informasi berdasarkan perda nomor L7/201, 6 tentang perlindungan lahan pertanian pangan berkelanjutan, ada 16 kecamatan yang menjadi percontohan program LP2B. Namun dalam realisasinya, hanya petani di 12 kecamatan yang sudah mendapatkan manfaat program tersebut. Pemkab Boyolali mengklaim 89% sawah lestari dengan sistem irigasi teknis sudah dibebaskan pajaknya hingga 90%. Pembebasan pajak ini menjadi bentuk insentif untuk mencegah adanya alih fungsi lahan pertanian.

Informasi yang dihimpun Espos di Badan Keuangan Daerah (BKD) Boyolali, Jumat (19/t), BKD telah mengukur lahan LP2B berdasarkan bidang yang terdaftar dalam Surat Pemberitahuan Pajak Terutang (SPPT), berbeda dengan Dinas Pertanian yang menentukannya berdasarkan luas lahan per hektar. Selisish yang tertera jumlah bidang dalam sertifikat dan jumlah bidang dalam SPPT terjadi karena beberapa hal. Di antaranya adalah perbedaan acuan pengukuran tanah antara sertifikat dan SPPT. Misalnya beberapa bidang tanah dalam sertifikat hanya dihimpun dalam satu SPPT sehingga jumlah SPPT lebih sedikit dari jumlah bidang sertifikat.

Sementara itu, terkait pajak tiap-tiap pemilik lahan dikenakan harga berbeda-beda dengan kisaran paling sedikit Rp 50.000 per tahun hingga ratusan ribu rupiah tergantung luas bidang. "Oleh karena itu, potensi pajak tiap petani tidak bisa digeneralisir," ujar kepala BKD, Agus Partono. Selisih jumlah antara bidang sertifikat dan bidang SPPT ini juga dipengaruhi bidang tanah yang masuk sebagai tanah Negara dan fasilitas umum sehingga tidak memiliki SPPT.

39. Berdasarkan paragraf pertama, mana simpulan di bawah ini yang BENAR?
A. Semua kecamatan di Boyolali belum tersentuh program insentif pajak bumi dan bangunan (PBB)
B. Semua kecamatan di Boyolali sudah tersentuh program insentif pajak bumi dan bangunan (PBB)
C. Sebagian kecamatan di Boyolali sudah tersentuh program insentif pajak bumi dan bangunan (PBB)
D. Sebagian program LP2B dilakukan di lahan sawah beririgasi teknis.
E. Sebagian kecamatan yang belum tersentuh program insentif pajak karena sedikitnya jumlah sawah yang bisa digunakan untuk menanam padi dan menggunakan irigasi teknis

Jawaban: E
Pembahasan: Simpulan adalah Sesuatu yang dikaitkan. Metode simpulan paragraf di atas adalah secara induktif yaitu terdapat di akhir paragraf.

40. Simpulan apa yang dapat ditarik dari paragraf tiga adalah ...
A. Jumlah bidang dalam sertifikat sama dengan jumlah bidang dalam SPPT
B. Ada selisih antara jumlah bidang dalam sertifikat dan jumlah bidang SPPT
C. BKD mengukur lahan LP2B berdasarkan luas tahan per hektar
D. Dinas pertanian mengukur lahan LP2B berdasarkan bidang yang terdaftar dalam Surat Pemberitahuan Pajak Terutang (SPPT)
E. Jumlah bidang sertifikat lebih sedikit dari jumlah SPPT

Jawaban: B
Pembahasan: Pembahasan: Simpulan adalah Sesuatu yang dikaitkan dan yang bekaitan hanya jawaban 'Ada selisih antara jumlah bidang dalam sertifikat dan jumlah bidang SPPT.

41. Berdasarkan paragraf 2, mana simpulan di bawah ini yang PALING 
MUNGKIN benar?
A. Ada 16 kecamatan yang sudah mendapatkan manfaat program LP2B
B. Berdasarkan perda nomor 17/201 tentang perlindunganlahan pertanian pangan berkelanjutan, ada 12 kecamatan yang menjadi percontohan program LP2B
C. Ada 4 kecamatan yang belum mendapatkan manfaat program LP2B
D. Pemkab Boyolali membebaskan pajak hingga 89% untuk sawah lestari dengan sistem irigasi teknis.
E. Pembebasan pajak bertujuan mendorong adanya alih fungsi lahan pertanian

Jawaban: E
Pembahasan: Jawaban yang paling mungkin benar adalah pembebasan pajak bertujuan mendorong adanya alih fungsi lahan pertanian (kalimat terakhir paragraf 2)

42. Dalam sebuah acara reuni sekolah, diketahui bahwa Irma lebih tua daripada Reni dan Dita lebih muda daripada Reni. Jadi ....
A. Irma lebih tua daripada Dital
B. Irma lebih muda daripada Dita
C. Reni lebih muda daripada Dita
D. Dita seusia dengan Reni
E. Irma seusia dengan Dita

Jawaban: A
Pembahasan: Dita adalah yang paling muda dari Reni dan Irma, sudah pasti jawabannya adalah Irma lebih tua daripada Dita.

43. Jika hujan lebat maka datang banjir.
Jika datang banjir maka banyak orang mengungsi.
Jadi ...
A. Jika tidak banjir maka tidak hujan lebat
B. Jika hujan lebat maka banyak orang mengungsi
C. Jika banyak orang mengungsi maka tidak hujan
D. Jika tidak ada yang mengungsi datang banjir
E. Tidak dapat disimpulkan

Jawaban: B
Pembahasan:
Jika hujan lebat = A
Datang banjir = B
Banyak orang mengungsi = C 
Pernyataan 1 = A-B 
Pernyataan 2 =B-C 
Simpulan = A-C

44. Semua penderita hipertensi tidak boleh banyak makan makanan asin. Kakek penderita hipertensi.
A. Semua makanan asin kakek suka
B. Sebagian penderita hipertensi boleh makan makanan asin.
C. Kakek tidak boleh banyak makan makanan asin.
D. Semua kakek tidak boleh makan makanan asin.
E. Tidak dapat ditarik kesimpulan.

Jawaban: C
Pembahasan: Karena semua tidak boleh, jadi kakek tidak boleh banyak makan makanan asin.

45. Dalam sebuah rumah makan disediakan beberapa varian menu ayam dengan harga yang bervariasi. Paket Ayam Pop 1 lebih mahal disbanding Paket Ayam Pop 2; Paket Ayam Pop 3 lebih mahal daripada Paket ayam Pop 2.
Mana dari pernyataan berikut yang benar....
A. Paket Ayam Pop 1 lebih mahal dibanding Paket Ayam Pop 3.
B. Paket Ayam Pop 3 lebih mahal dibanding Paket Ayam Pop 1.
C. Harga Paket Ayam Pop 1 ditambah harga Paket Ayam Pop 2 lebih besar dibanding harga Paket Ayam Pop 1 ditambah harga Paket Ayam Pop 3.
D. Harga Paket Ayam Pop 2 lebih kecil dibanding rata-rata harga Paket Ayam Pop 1 dan harga Paket Ayam Pop 3.
E. Tidak ada pernyataan yang sesuai.

Jawaban: D
Pembahasan: Karena yang paling murah adalah paket ayam pop 2 jadi Harga Paket Ayam Pop 2 lebih kecil dibanding rata-rata harga Paket Ayam Pop 1 dan harga Paket Ayam Pop 3.

46. Sebagian makanan di pasar harganya murah 
Semua makanan di pasar rasanya enak
A. Semua yang di pasar enak
B. Tidak ada makanan yang tak enak
C. Semua makanan di pasar harganya tidak murah
D. Sebagian yang rasanya enak harganya murah
E. Tidak satu pun yang harganya murah itu enak

Jawaban: D
Pembahasan: Oleh karena semua makanan di pasar enak dan hanya sebagian yang harganya murah, jadi sebagian yang rasanya enak harganya murah.

47. Semua muslimah berjilbab 
Sebagian muslimah sudah naik haji
A. Sebagian yang sudah naik haji berjilbab
B. Semua yang berjilbab sudah naik haji
C. Tidak ada haji yang berjilbab
D. Semua muslimah sudah naik haji
E. Sebagian haji muslimah yang tidak berjilbab

Jawaban: A
Pembahasan: Yang tepat adalah Sebagian yang sudah naik haji berjilbab
Dalam suatu kelas terdapat 12 murid laki-laki dan 16 murid perempuan. Rata-rata nilai ulangan Matematika di kelas tersebut adalah 80. Setelah melihat hasil tersebut, guru Matematika memberikan kesempatan kepada 4 murid, dengan nilai masing-masing 52, 56, 62, dan 66, untuk melakukan remedial. Diketahui bahwa nilai rata-rata peserta remedial naik 7 poin.

48. Jika sebelum remedial, rata-rata nilai ulangan murid laki-laki di kelas 
tersebut adalah 78, rata-rata nilai ulangan murid perempuan adalah ...
A. 80,5
B. 81
C. 81,5
D. 82
E. 82,5

Jawaban: C 
Pembahasan:
Jumlah total nilai ulangan adalah 80x28 = (a1+...+a12) + (b1+...+b16) = 12x78 + (b1+...+b16).
Jadi, nilai rata-rata ulangan murid perempuan adalah (b1+...+b16)/16 = (80x28-12x78)/16 = 81,5.

49. Untuk bilangan bulat a, b dan c didefinisikan a*b*c = a + b. Nilai dari 1*2*3 adalah....
A. 7
B. 8
C. 9
D. 10
E. 11

Jawaban: C 
Pembahasan: 
a*b*c = a + b2 
1*2*3 =1+23 
=9

50. Faktor persekutuan terbesar setiap 2 bilangan diantara bilangan asli a, b, dan c adalah 1. Jika a(b-1)=24, 2a - c = 5 dana < 6
(i) Hasil ke 3 bilangan adalah 84
(ii) Selisih 2 bilangan terbesar adalah 3
(iii) Selisih bilangan terbesar dan terkecil adalah 4
(iv) Bilangan terkecil adalah lima 
maka pernyataan yang benar adalah...
A. (i) dan (iii)
B. (ii) dan (iv)
C. (i), (ii), dan (iii)
D. (iv)
E. Semua benar

Jawaban: C 
Pembahasan: 
a<6
a = 1, 2, 3, 4 
a=4
a(b-1) = 24 
4(b-1) = 24 
b=7
2a-c=5 
c=3
(i) 3x4x7 = 84
(ii) 7-4=3
(iii) 7-3 = 4
(iv) Bilangan terkecil adalah 5

51. Suatu bangunan merupakan gabungan dari 10 persegi identik yang tidak tumpeng tindih. Jika luas bangunan tersebut 250 maka kelilingnya adalah....
A. 90
B. 100
C. 110
D. 120
E. 200

Jawaban: C 
Pembahasan: 
10 x^2 = 250 
X=5
5 X 22 = 110

52. Jika f(x)=2x-3 dan (gof)(x)= 2x+1, maka g(x)=....
A. x+1
B. x+2
C. x+3
D. x+4
E. x+5

Jawaban: D 
Pembahasan: 
g (f(x) = 2x + 1 
g (2x-3)=(2x-3)+4 
g(x) = x + 4

53. Jika titik A (2, -4) digeser sejauh 4 satuan ke kiri dan 2 satuan ke 
atas, lalu dicerminkan ke sumbu-y, maka bayangannya adalah A'. Jarak A 
ke A' adalah.... satuan.
A. 1
B. 2
C. 3
D. 4
E. 5

Jawaban: B 
Pembahasan:
A (2,-4) digeser 4 satuan ke kiri dan 2 satuan ke atas menjadi (2-4,-4+2)= (-2, -2).
Bila (-2, -2) dicerminkan ke sumbu-y, hasilnya (-2x (-1),-2) = A' (2,-2). 
Jarak A(2, -4) ke A' (2, -2) adalah 2 satuan.

54. Dari 5 angka 1, 2, 3, 4 dan 5 akan dibentuk bilangan yang terdiri dari 4 angka. Berapa bilangan yang dapat disusun jika bilangan tersebut kurang dari 3000 dan angka yang digunakan tidak boleh berulang?
A. 30
B. 48
C. 60
D. 72
E. 120

Jawaban: B 
Pembahasan:
Angka yang dapat menempati tempat ribuan ada 2 angka (karena kurang 
dari 3000 maka yang dapat menempati tempat ribuan adalah angka 1 
dan 2);
Angka yang dapat menempati tempat ratusan ada 4 angka (karena 1 angka sudah menempati ribuan); Angka yang dapat menempati tempat puluhan ada 3 angka (karena 2 angka sudah menempati tempat ribuan dan ratusan);
Angka yang dapat menempati tempat satuan ada 2 angka (karena 3 angka sudah menempati tempat ribuan, ratusan dan puluhan); 
Banyak bilangan yang dapat dibentuk ada 2 x 4 x 3 x 2 = 48 bilangan.

Perhatikan ilustrasi berikut untuk menjawab soal nomor 1-3! 
Anggun membawa tiga buah barang, yaitu A, B, dan C yang beratnya berturut-turut 1.450 g, 0,5 kg, dan 3,2 kg ke WahAda Cargo untuk dikirimkan ke Cimahi. Berikut ini tabel yang menunjukkan biaya pengiriman barang.

Berat (g) Biaya (Rp)
<500 3.600
500 - 1000 6.900
1.001 - 3.5000 13.250
3.501 - 5.000 24.500
> 5.000 37.000

55. Jika Anggun ingin berhemat, yang paling menguntungkan bagi Anggun di antara pilihan berikut ini adalah ...
A. Pengiriman barang A, B, dan C masing-masing terpisah.
B. Pengiriman barang A dan B digabung, sedangkan barang C terpisah.
C. Pengiriman barang A dan C digabung, sedangkan barang B terpisah.
D. Pengiriman barang B dan C digabung, sedangkan barang A terpisah.
E. Pengiriman barang A, B, dan C digabung.

Jawaban: B 
Pembahasan:
Diketahui bahwa barang A seberat 1.450 g barang B seberat 0,5 kg = 500 g dan barang C seberat 3,2 g = 3.200 g.
Untuk mengetahui pilihan yang paling menguntungkan, akan diperiksa  biaya yang dikeluarkan pada setiap pilihan jawaban yang diberikan.
Pilihan jawaban A: Pengiriman barang A, B, dan C masing-masing 
terpisah.
Karena ketiga barang dikirim terpisah, maka total biaya yang dikeluarkan adalah sebagai berikut.
Total biaya
= biaya barang A + biaya barang B + biaya barang C 
= 13.250 + 6.900 + 13.250
= 33.400
Didapat bahwa biaya pengiriman pada pilihan A adalah Rp33.400,00. 
Pilihan jawaban B: Pengiriman barang A dan B digabung, sedangkanbarang C terpisah.
Karena barang A digabung dengan barang B, maka beratnya menjadi 1.450g + 500g = 1.950g. Kemudian total biaya yang dikeluarkan adalah sebagai berikut.
Total biaya
= biaya barang A + biaya barang B + biaya barang C 
= 13.250 + 13.250
= 26.500
Didapat bahwa biaya pengiriman pada pilihan B adalah Rp26.500,00. 
Pilihan jawaban C: Pengiriman barang A dan C digabung, sedangkan
barang B terpisah.
Karena barang A digabung dengan barang C, maka beratnya menjadi 1.450g + 3.200g = 4.650g. Kemudian, total biaya yang dikeluarkan  adalah sebagai berikut.
Total biaya
= biaya barang A dan C + biaya barang B 
= 24.500 + 6.900
= 31.400
Didapat bahwa biaya pengiriman pada pilihan C adalah Rp31.400,00. 
Pilihan D: Pengiriman barang B dan C digabung, sedangkan barang Aterpisah.
Karena barang B digabung dengan barang C, maka beratnya menjadi 500g + 3.200g = 3.700g. Maka, total biaya yang dikeluarkan adalah sebagai berikut.
Total biaya
= biaya barang B dan C + biaya barang A 
= 24.500 + 13.250
= 37.750
Didapat bahwa biaya pengiriman pada pilihan D adalah Rp37.750,00. 
Pilihan E: Pengiriman barang A, B, dan C digabung.
Karena semua barang digabung, maka beratnya menjadi 1.450g + 500g + 3.200g = 5.150g. Kemudian, total biaya yang dikeluarkan adalah sebagai berikut.
Total biaya
= biaya barang ABC 
= 37.700
Didapat bahwa biaya pengiriman pada pilihan E adalah Rp37.700,00. 
Berdasarkan perhitungan tersebut, biaya paling murah adalah Rp26.500,00.
Dengan demikian, jika Anggun ingin berhemat, yang paling menguntungkan bagi Anggun adalah pengiriman barang A dan B digabung, sedangkan barang C terpisah. Jadi, jawaban yang benar adalah B.

56. Diketahui terdapat penambahan biaya pengemasan sebesar Rp3.000,00 untuk setiap pengiriman. Jika Anggun ingin mengirim dua barang sekaligus dan satu barang terpisah, biaya termurah yang dikeluarkan Anggun adalah..
A. Rp44.650,00
B. Rp43.750,00
C. Rp37.400,00
D. Rp32.500,00
E. Rp20.150,00

Jawaban: D 
Pembahasan:
Diketahui bahwa barang A seberat 1.450 g barang B seberat 0,5 kg = 500 g dan barang C seberat 3,2 g = 3.200 g.
Diketahui pula terdapat penambahan biaya pengemasan sebesar  Rp3.000,00 untuk setiap pengiriman.
Untuk mengetahui total biaya termurah yang dikeluarkan oleh Anggun, akan diperiksa biaya yang dikeluarkan untuk setiap kemungkinan berikut.
Kemungkinan 1: Pengiriman barang A dan B digabung, sedangkan barang C terpisah.
Karena barang A digabung dengan barang B, maka beratnya menjadi 1.450g + 500g = 1.950g. Total biaya yang dikeluarkan adalah sebagai berikut.
Total biaya
= Biaya barang A dan B + biaya barang C + 2 x biaya pengemasan 
= 13.250 + 13.250 + 3.000 x 2
= 26.500 + 6.000 
= 32.500
Didapat bahwa biaya yang dikeluarkan Anggun pada kemungkinan 2 adalah Rp 32.500,00. Kemungkinan 2: Pengiriman barang A dan C digabung, sedangkan barang B terpisah. Karena barang A digabung dengan barang C, maka beratnya menjadi 1.450g + 3.200g = 4.650g. Total biaya yang dikeluarkan adalah sebagai berikut. Total biaya = Biaya barang A dan C + biaya barang B + 2 x biaya pengemasan 
= 24.500 + 6.900 + 3.000 x 2
= 31.400 + 6.000 
= 37.400
Didapat bahwa biaya yang dikeluarkan Anggun pada kemungkinan 1 adalah Rp37.400,00.
Kemungkinan 3: Pengiriman barang B dan C digabung, sedangkan  barang A terpisah. Karena barang B digabung dengan barang C, maka beratnya menjadi 500g + 3.200g = 3.700g. Total biaya yang dikeluarkan adalah sebagai berikut. Total biaya = Biaya barang B dan C + biaya barang A + 2 x biaya pengemasan  = 24.500 + 13.250 + 3000 x 2 = 37.750 + 6.000  = 43.750 Didapat bahwa biaya yang dikeluarkan Anggun pada kemungkinan 3 adalah Rp 43.750,00. Berdasarkan ketiga kemungkinan tersebut, biaya termurah yang dikeluarkan Anggun adalah Rp32.500,00.
Jadi, jawaban yang benar adalah D

Teks ini digunakan untuk menjawab soal nomor 57-58.
(1) Sebagian besar orang sering mengeluh karena terlalu sibuk. (2) Mereka umumnya ingin memiliki lebih banyak waktu luang. (3) Namun, penelitian terbaru menemukan bahwa terlalu banyak waktu luang ternyata tidak lebih baik daripada terlalu sibuk. (4) Menurut penelitian yang diterbitkan oleh American Psychological Association, bertambahnya waktu luang memang dapat meningkatkan rasa bahagia. (5) Akan tetapi, perasaan itu hanya bertahan sampai titik tertentu. (6) Jika waktu luang yang dimiliki terlalu banyak, akan ada dampak buruk yang timbul.

(7) Untuk menyelidiki fenomena tersebut, para peneliti melakukan eksperimen daring yang melibatkan lebih dari 6.000 peserta. (8) Peneliti menemukan bahwa orang yang memiliki waktu luang sedikit merasa lebih stres daripada mereka yang memiliki jumlah waktu luang sedang. (9) Sementara itu, mereka yang memiliki waktu luang banyak juga merasa kurang produktif daripada mereka yang berada dalam kelompok sedang. (10) Lebih lanjut, temuan tersebut menunjukkan bahwa berakhir  dengan waktu luang sepanjang hari untuk melakukan hal-hal yang diinginkan ternyata dapat membuat seseorang merasa tidak bahagia. (11) Sebaliknya, orang harus berusaha untuk memiliki waktu luang dalam jumlah sedang agar dapat melakukan apa yang mereka inginkan.

57. Topik bacaan tersebut adalah ....
A. perbandingan antara orang yang memiliki waktu luang dengan orang yang sibuk
B. kelebihan dan kekurangan dari adanya waktu luang yang terlalu banyak
C. memiliki terlalu banyak waktu luang tidak lebih baik daripada terlalu sibuk
D. dampak buruk yang dialami oleh orang-orang yang memiliki waktu luang
E. Penelitian American Psychological Association tentang kesibukan dan waktu luang

Kunci Jawaban: C
Pembahasan: Topik dari sebuah bacaan sama halnya dengan ide pokok atau gagasan utama bacaan tersebut. Untuk dapat menemukannya, pembaca perlu memahami ide pokok ataupun informasi penting dalam setiap paragraf, kemudian menyimpulkan inti bacaan berdasarkan hal-hal tersebut.
Bacaan pada soal tersebut terdiri dari dua paragraf. Paragraf 1 menyebutkan bahwa terlalu banyak waktu luang tidak lebih baik daripada terlalu sibuk. Dalam paragraf 1, disebutkan pula adanya dampak buruk 
dari waktu luang yang terlalu banyak. Sementara itu, paragraf 2 membahas penelitian tentang jumlah waktu luang yang terbaik untuk dimiliki oleh setiap orang. Dalam paragraf 2, dijelaskan bahwa memiliki waktu luang yang terlalu banyak atau terlalu sedikit tidak baik. Jumlah waktu luang yang paling ideal adalah jumlah yang sedang. Berdasarkan isi kedua paragraf, dapat disimpulkan bahwa topik bacaan tersebut adalah terlalu banyak waktu luang tidak lebih baik daripada terlalu sibuk.

58. Makna yang sama dari kata dampak pada kalimat (6) terdapat pula 
pada kata ....
A. impak
B. efek
C. imbas
D. akibat
E. implikasi

Kunci Jawaban: A
Pembahasan: Setiap kata memiliki maknanya masing-masing. Selain itu, ada pula kata-kata tertentu yang memiliki makna yang sama dengan kata lain. Kata yang bermakna sama dengan kata lain disebut sinonim. Dalam Kamus Besar Bahasa Indonesia (KBBI), kata dampak memiliki tiga arti, yakni (1) 'benturan', (2) 'pengaruh kuat yang mendatangkan akibat (baik negatif maupun positif)', dan (3) 'benturan yang cukup hebat antara dua benda sehingga menyebabkan perubahan yang berarti dalam momentum (pusa) sistem yang mengalami benturan itu'. Dalam bacaan, kata dampak muncul pada kalimat (6) yang berbunyi Jika waktu luang yang dimiliki terlalu banyak, akan ada dampak buruk yang timbul. Kata dampak pada kalimat tersebut bermakna 'pengaruh kuat yang mendatangkan akibat'. Kata tersebut bersinonim atau bermakna sama dengan kata impak yang dapat berarti 'pengaruh yang kuat; dampak'.

Teks ini digunakan untuk menjawab soal nomor 59-60.
Nelayan yang berdampak reklamasi Terminal Petikemas pelabuhan Belawan Medan meminta ganti rugi kepada PT Pelabuhan Indonesia I. Nelayan rugi karena reklamasi menutup alur kapal nelayan sehingga mereka harus menambah 1 mil perjalanan setiap hari dengan tambahan 1 liter solar. Perwakilan nelayan terdampak menyampaikan hal tersebut pada rapat dengar pendapat di Dewan Perwakilan Rakyat Daerah Sumatera Utara, Medan. Reklamasi Pelabuhan Belawan sudah berlangsung tiga tahun. Namun, sampai sekarang, nelayan belum mendapat ganti rugi. Rapat yang dipimpin Ketua Komisi B DPRD Sumut itu dihadiri Kepala Bidang Perikanan Tangkap Dinas Pertanian dan Perikanan Kota Medan, Friska Irnawati Purba, serta Kepala Bidang Perencanaan dan Pembangunan Kantor Otoritas Pelabuhan Utama Belawan, Thomas. Octavianus Sipa helut. PT Pelabuhan Indonesia I diundang, tetapi tidak hadir.

Selama tiga tahun mereka belum mendapat ganti rugi. Baru pada 10 April, ada 3.228 nelayan dinyatakan sudah diverifikasi dan akan menerima tali kasih Rp 3.097.000 pernelayan. Delapan orang menerima dana secara simbolis. Sebanyak 300 nelayan tidak masuk dalam daftar nelayan yang terdampak reklamasi yang diverifikasi. Para nelayan hanya minta ganti rugi untuk tambahan bahan bakar selama empat tahun, sekitar 6 juta per nelayan. Petugas Pertanian dan Perikanan Kota Medan kesulitan melakukan verifikasi karena ada penolakan nelayan di lapangan. Akibatnya, sejumlah nelayan yang tidak masuk dalam daftar nelayan yang terdampak. Data hasil verifikasi terhadap 3.228 nelayan sudah final dan tidak ada tambahan lagi.

59. Kalimat inti kalimat pertama paragraf kedua teks tersebut adalah ....
A. Nelayan terdampak.
B. Nelayan menyampaikan hal tersebut.
C. Perwakilan nelayan mendengarkan pendapat.
D. Perwakilan nelayan menyampaikan hal tersebut.
E. Nelayan dengar pendapat.

Jawaban: D
Pembahasan: Yang harus dilakukan untuk menemukan kalimat inti atau inti kalimat adalah menemukan subjek dan predikat. 
Caranya:
Hilangkan semua perluasan (yang, mengenai, yakni, yaitu, seperti, dll) 
Hilangkan semua keterangan.
Jika kalimat aslinya berupa kalimat majemuk, kalimat intinya berupa 
klausa utama atau klausa pokok kalimat tersebut. Perwakilan nelayan 
terdampak menyampaikan hal tersebut pada rapat dengar pendapat di 
Dewan Perwakilan Rakyat Daerah sumatera Utara, Medan.

60. Topik bacaan tersebut adalah....
A. Keadaan nelayan reklamasi.
B. Realisasi ganti rugi nelayan reklamasi.
C. Dampak reklamasi.
D. Solusi bagi nelayan reklamasi.
E. Hak nelayan reklamasi.

Jawaban: B
Pembahasan: Topik bacaan sama seperti menemukan kalimat utama. 
Karena yang ditanyakan adalah topik keseluruhan isi bacaan, kita harus membaca seluruhnya dan menemukan hal yang diulang atau yang dibahas terus-menerus. Topik bacaan ini adalah realisasi ganti rugi nelayan reklamasi.

61. Pernyataan berikut yang paling benar dari teks tersebut adalah ....
A. Realisasi ganti rugi yang diberikan sudah sesuai dengan tuntutan nelayan sekalipun belum terealisasi dengan baik.
B. Seluruh nelayan yang telah masuk verifikasi akan segera mendapat ganti rugi reklamasi.
C. Reklamasi Pelabuhan Belawan dinilai tidak efektif dan menimbulkan keresahan di masyarakat.
D. Nelayan meminta ganti rugi bahan bakar dan biaya melaut selama empat tahun akibat dampak reklamasi.
E. Sejumlah nelayan tidak masuk daftar nelayan terdampak karena adanya penolakan di lapangan.

Jawaban: E
Pembahasan: Pernyataan yang paling benar berarti yang paling sesuai dengan isi teks tersebut. Pernyataan yang lain kemungkinan juga benar tapi pembaca diminta menemukan satu jawaban yang paling tepat. Pernyataan tersebut terdapat di kalimat terakhir.

62. Simpulan paragraf kedua teks tersebut adalah.....
A. Nelayan menyambut positif hasil rapat tersebut.
B. Dengan adanya rapat tersebut, nelayan optimis masalah mereka akan segera teratasi.
C. Jadi, rapat tersebut adalah salah satu solusi yang dinanti-nantikan nelayan.
D. Oleh karena itu, harus ada realisasi nyata dari rapat yang telah dilangsungkan tersebut.
E. Hasil rapat tersebut dinilai menguntungkan nelayan dan merupakan solusi yang terbaik dari semua pihak.

Jawaban: D
Pembahasan: Simpulan paragraf berarti hanya mengacu pada paragraf tersebut itu saja dan harus mencakup keseluruhan isi paragraf. Simpulan paragraf kedua teks tersebut ada di pilihan jawaban D

63. Kesalahan Ejaan dalam kalimat tersebut ditemukan dalam kalimat
A. Kalimat kedua paragraf ketiga.
B. Kalimat ketiga paragraf kedua.
C. Kalimat kelima paragraf keempat.
D. Kalimat kesatu paragraf kedua.
E. Kalimat keempat paragraf kedua.

Jawaban: A
Pembahasan: Kesalahan ejaan meliputi pemakaian tanda baca yang salah, penulisan huruf, serta penulisan kata dan angka. Kesalahan penulisan ejaan di kalimat tersebut ditemukan di kalimat (2) paragraf (3) tulisan pernelayan harus dipisah menjadi per nelayan. Penulisan per yang digabung jika menyatakan tiap (contoh: per minggu), demi (contoh: satu per satu), dan mulai (contoh 1 Desember 2017).

64. Salah satu kalimat dalam paragraf tersebut tidak memenuhi unsur sebuah kalimat. Kalimat yang dimaksud adalah ....
A. Kalimat kedua paragraf ketiga.
B. Kalimat ketiga paragraf kedua.
C. Kalimat kelima paragraf kesatu.
D. Kalimat kesatu paragraf kedua.
E. Kalimat kedelapan paragraf ketiga.

Jawaban: C
Pembahasan: Unsur mutlak sebuah kalimat harus terdiri atas subjek dan predikat. Kalimat (8) paragraf (3) bacaan tersebut tidak mempunyai predikat karena diperluas dengan konjungsi yang.

65. Hubungan atau keterikatan emosional antara orang tua dan anak adalah anugerah alami yang dimiliki setiap orang tua dalam mengasuh dan membesarkan anak-anak mereka. Orang tua secara alamiah telah melakukan tugas dan fungsi pengasuhan anak sejak peran sebagai orang tua dimulai. Akan tetapi, dalam realitasnya tetap diperlukan pengetahuan dalam khazanah pengasuhan anak remaja agar mereka tidak terlibat dalam penyalahgunaan narkotika dan obat-obat terlarang. 
Simpulan yang tepat untuk paragraf di atas adalah...
A. Orang tua berkewajiban membesarkan anak-anak mereka.
B. Orang tua memerlukan bekal pengetahuan dalam mengasuh anak.
C. Anak dan orang tua dalam pengasuhan memiliki hubungan emosional.
D. Keterikatan emosional antara anak dan orang tua penting.
E. Orang tua memiliki peran penting dalam mengasuh anak sejak dini.

Jawaban: D 
Pembahasan:
Umumnya, simpulan memuat pendapat. 
Kalimat utama:
Hubungan atau keterikatan emosional antara orang tua dan anak adalah anugerah alami yang dimiliki setiap orang tua dalam mengasuh dan membesarkan anak-anak mereka. Inti pembicaraan pada kalimat utama adalah mengenai hubungan atau keterkaitan emosional. 
Jadi, simpulan yang tepat untuk paragraf tersebut adalah keterikatan emosional antara anak dan orang tua penting.

66. Sirine sebagai bagian dari sistem peringatan dini tsunami dijalankan dengan menggunakan teknologi komunikasi satelit dan GSM. Begitu ada gempa, sensor yang dipasang pada sirine akan memberikan data secara cepat ke stasiun BMG. Berdasarkan data ini, dilakukan analisis untuk mengetahui terjadinya tsunami atau tidak dalam waktu 10 menit. Bila ada potensi akan terjadi tsunami, sirine langsung dibunyikan dalam bentuk nada tertentu, disertai pengumuman kepada penduduk setempat agar mereka segera mempersiapkan diri dan mengambil tindakan preventif. 
Topik bacaan di atas adalah....
A. proses penanganan tsunami secara dini
B. proses penanggulangan terjadinya tsunami
C. proses kerja sistem peringatan dini tsunami
D. proses kerja sirine sebagai sistem peringatan dini tsunami
E. proses penanganan penduduk yang tinggal di daerah rawan tsunami

Jawaban: D
Pembahasan: topic merupakan inti dari bacaan atau teks maka topic yang tepat adalah proses kerja sirine sebagai sistem peringatan dini tsunami

67. Akibatnya Indonesia tidak bisa membeli persenjataan dari Amerika serikat. Selain itu, Indonesia juga tidak bisa membeli suku cadang untuk perawatan peralatan dari Amerika Serikat. Sebagai contoh, dari belasan pesawat Hercules yang dimiliki Indonesia hanya kurang dari lima pesawat saja yang bisa digunakan.
Untuk melengkapi paragraf tersebut menjadi paragraf deduktif, kalimat utama yang tepat adalah..
A. Amerika Serikat siap mencabut embargo militer terhadap Indonesia.
B. Pencabutan embargo militer terhadap Indonesia disebabkan maraknya aktivitas terorisme di
Indonesia.
C. Amerika Serikat mengenakan embargo senjata pada Indonesia karena banyak warganya menjadi korban pemboman di Indonesia.
D. Presiden Amerika Serikat akan mencabut embargo militer terhadap pemerintah Indonesia.
E. Embargo militer dilakukan Amerika Serikat terhadap Indonesia karena kerusuhan Santa Cruz, Dili, pada 1991.

Jawaban: C
Pembahasan: Karena Indonesia tidak bisa membeli persenjataan dan suku cadang maka Indonesia dikenakan embargo senjata.

68. Kanker paru saat ini sangat ditakuti sebagai salah satu penyakit yang mematikan. Hanya 25 persen kanker jenis ini bisa terdiagnosis pada tahap dini. Para ahli telah berupaya menemukan cara agar bisa mendeteksi sedini mungkin adanya kanker paru sehingga pasien bisa diselamatkan. Di antara mereka adalah para peneliti dari University of Kentucky's Chandler Medical di Lexington, AS, yang mencoba teknik pemeriksaan antibody dengan ketepatan tinggi untuk mendiagnosis sel kanker tahap dini. Mereka menganalisis 212 "calon" penderita kanker dengan semacam protein antibody. Ternyata protein tersebut 90 persen akurat. Cara ini bisa digunakan untuk membedakan antara tumor jinak dan kanker.

Paragraf di atas menjadi lengkap jika dipadukan dengan kalimat penutup sebagai berikut....

A. Karena itu, protein banyak dipakai untuk mendeteksi tumor jinak dan kanker.
B. Oleh sebab itu, kanker paru saat ini tidak perlu ditakuti lagi karena telah ditemukan protein.
C. Akhirnya, untuk mendiagnosis sel kanker hanya dibutuhkan semacam protein.
D. Dengan demikian, untuk mendeteksi kanker sedini mungkin, para ahli menggunakan protein antibodi.
E. Jadi, penderita kanker dapat mendeteksi secara dini dengan bantuan teknik antibodi.

Jawaban: D
Pembahasan: kalimat penutup yang tepat adalah Dengan demikian, untuk mendeteksi kanker sedini mungkin, para ahli menggunakan protein antibodi.

69. Pemanasan global yang memicu perubahan iklim tidak terelakkan dan sedang berlangsung. Penyebabnya adalah penggunaan bahan bakar fosil yang terus menerus menyemburkan emisi melalui cerobong dan knalpot. Para ilmuwan memperkirakan bahwa pada abad ini terjadi peningkatan suhu global antara 1,4 dan 5,8 derajat celcius bergantung pada volume bahan bakar fosil yang dibakar dan kepekaan system iklim menghadapinya. Bahan bakar fosil seperti minyak, batu bara, dan gas sebagai sumber energi menghasilkan gas CO2 yang dilepaskan ke atmosfer sebagai sisa pembakaran. Gas tersebut beserta gas-gas lain menumpuk, menyelimuti bumi, dan menghalangi radiasi panas matahari yang seharusnya dikembalikan ke angkasa. Radiasi panas dipantulkan kembali ke bumi, diserap di permukaan, dan menimbulkan fenomena yang disebut efek gas rumah kaca. Efek gas rumah kaca yang terjebak di atmosfer itulah yang meningkatkan secara drastis temperatur di permukaan bumi sehingga terjadi pemanasan global. Penggunaan bahan fosil secara besar-besaran telah menyumbangkan 37 persen dari total emisi CO2 di Indonesia. Angka tersebut akan terus meningkat jika tidak ada upaya menghentikan atau mengurangi penggunaan energi fosil.

Berdasarkan isi teks tersebut, pernyataan ini benar, KECUALI
A. Efek gas rumah kaca terjadi karena radiasi panas matahari di bumi tidak dapat dikembalikan ke angkasa akibat terhalang gas CO2.
B. Menurut perhitungan para ilmuwan, temperatur udara di bumi naik dari 1,4 sampai 5,8 derajat Celcius setiap abad.
C. Pemanasan global dapat dikendalikan dengan mengurangi pemakaian energi fosil.
D. Pemakaian bahan bakar fosil menghasilkan gas CO2 yang dapat menghalangi radiasi panas matahari kembali ke angkasa.
E. Lebih dari sepertiga total emisi CO2 di Indonesia disebabkan oleh pemakaian bahan bakar fosil.

Jawaban: B
Pembahasan: Pernyataan yang benar adalah Para ilmuwan memperkirakan bahwa pada abad ini terjadi peningkatan suhu global antara 1,4 dan 5,8 derajat celcius.

70. Bentuk akronim terdapat dalam kalimat ...
A. Sidang tahunan MPR berlangsung dari tanggal 1 sd 10 Agustus 2002.
B. Ibu membeli 5 kg gula dan 2 I minyak goreng.
C. Seorang satpam berdiri di muka kantor itu.
D. Dulu dia bekerja di RSPAD Jakarta.
E. Dulu kami tinggal di RT 007 RW 011 Kelurahan Rawamangun

Jawaban: C
Pembahasan: Akronim adalah kependekan yang berupa gabungan huruf atau suku kata atau bagian lain yang ditulis dan dilafalkan sebagai kata yang wajar, maka jawaban yang tepat adalah Seorang satpam berdiri di muka kantor itu.

71. Kain adat Ulos, Uis Karo, dan Uis Beka yang dirajut dengan alat tenun tradisional gedokan, banyak diburu oleh karena halus dan rapi. Penenun yang menggunakan gedokan tidak bisa bebas bergerak karena alat itu harus dipangku pada saat dipakai, kemampuannya hanya satu lembar kain setiap kali menenun. Sahat, penenun pribumi, mencoba memberanikan diri memakai Alat Tenun Bukan Mesin (ATBM) untuk merajut kain adat. Ternyata hasil rajutannya tidak kalah halusnya dengan alat tradisional gedokan. Di samping halus dan rapi, hasilnya dapat meningkatkan nilai produksinya. Dialah yang menepis anggapan bahwa kain adat
hanya dirajut dengan menggunakan gedokan. 

Simpulan dari penggalan berita tersebut adalah...
A. Kain adat tradisional Ulos, Uis Karo, dan Uis Beka banyak diburuorang.
B. Alat tenun gedokan sebaiknya diganti dengan ATBM.
C. Kain adat dapat dirajut melalui ATBM.
D. Pecinta seni hanya memburu kain adat yang dirajut dengan gedokan.
E. Penenun gedokan harus bekerja lebih keras daripada penenun ATBM.

Jawaban: A
Pembahasan: Jawaban yang tepat adalah Kain adat tradisional Ulos orang.

72. Sejak tahun 1980-an saat film Indonesia mulai tampil di panggung perfilman internasional, dalam pembuatan film yang bermutu menjadi salah satu tuntutan. Para pekerja film tergugah ingin menguasai pengetahuan tentang pembuatan film. Para pekerja film tergugah ingin menguasai pengetahuan tentang pembuatan film. Sehingga mereka mulai menyadari bahwa kelemahan film Indonesia terletak pada skenarionya. Mengenai kelemahan itu bukanlah disebabkan oleh penulis skenario yang kurang berbakat, melainkan kurangnya penguasaan teknik atau teori penulisan skenario. Sebab sebuah film dapat menjadi baik kalau dirancang dengan skenario yang bagus sehingga jelas jalan ceritanya.

Kalimat-kalimat dalam paragraph tersebut tidak baku, sehingga perlu diperbaiki dengan cara berikut,kecuali ...
A. menghilangkan kata dalam pada kalimat pertama
B. menghilangkan kata bahwa pada kalimat ketiga.
C. menggabungkan kalimat kelima dengan kalimat keempat.
D. menggabungkan kalimat ketiga dengan kalimat kedua.
E. menghilangkan kata mengenai pada kalimat keempat.

Jawaban: E
Pembahasan: Jawaban yang tepat adalah menghilangkan kata mengenai pada kalimat keempat.

73. Mitigasi bencana ada dua macam, yaitu mitigasi struktural dan nonstrktural. Mitigasi struktural biasanya terkait dengan bangunan tahan gempa, sedangkan yang nonstruktural berhubungan dengan pendidikan dan tata guna lahan.

Padanan kata yang tepat untuk kata mitigasi dalam kalimat tersebut 
adalah ....
A. Peringanan
B. Peringatan
C. Pengurangan
D. Peredaan
E. Pelonggaran

Jawaban: C
Pembahasan: Arti mitigasi adalah tindakan mengurangi dampak bencana.

(1)Teknologi modifikasi cuaca [...] beberapa hari terakhir di daerah tangkapan air Danau Toba. (2)Curah hujan mulai meningkat, tetapi tinggi muka air Danau Toba masih terus menurun. (3)[...] tinggi muka air [...] banyak dermaga tidak bisa digunakan, PLTA terancam berhenti beroperasi, irigasi terganggu, pasokan air baku rumah tangga dan industri berkurang, serta pariwisata terancam. (4)Level tinggi muka air (TMA) Danau Toba per Selasa (6/4/2021) pun sudah [...] 903,175 meter di atas permukaan laut (mdpl). (5)"Teknologi modifikasi cuaca diterapkan untuk [...] penurunan TMA Danau Toba agar tidak sampai ke level batas minimum, yakni 902,40 mdpl", kata Koordinator Lapangan Teknologi Modifikasi Cuaca Daerah Aliran Sungai (DAS) Danau Toba Cornelius A Nababan, Rabu (7/4/2021). (6)Teknologi modifikasi cuaca dilakukan dengan mensemprotkan bahan semai flare cosat berupa kalsium klorida dan magnesium klorida dari bawah atau atas awan. (7)Metode ini berbeda dengan [...] garam (NaCl) ke punggung awan yang biasa digunakan dalam menangani kebakaran hutan.

74. Kata yang tepat untuk mengisi bagian yang rumpang pada teks di 
atas adalah...
A. Digunakan, menjadikan, berada, mencegah, ditabur.
B. Diterapkan, membuat, berada, tahanan, taburkan
C. Diterapkan, membuat, berada, menahan, penyemaian
D. Digunakan, membuat, keberadaan, menahan, menabur
E. Diterapkan, menjadikan, berada, menahan, persemaian

Jawaban: C
Kata berimbuhan yang tepat sesuai konteksnya adalah kata diterapkan, membuat,berada, menahan, penyemaian.

75. Asas paralelisme atau kesejajaran bentuk pada teks di atas 
ditemukan pada...
A. Kalimat 2
B. Kalimat 3
C. Kalimat 4
D. Kalimat 5
E. Kalimat 6

Jawaban: E
Kesejajaran bentuk ditemukan pada kalimat 6, yaitu ...bawah (KS) atau atas (KS)

76. Kesalahan penggunaan ejaan pada teks di atas terdapat pada 
kalimat...
A. (1) dan (2)
B. (3) dan (4)
C. (1) dan (4)
D. (6) dan (7)
E. (3) dan (5)

Jawaban: E
Kalimat 3 > sesudah kata rumah tangga seharusnya diberi tanda koma (rincian)
Kalimat 5 > tanda koma sesudah kalimat langsung seharusnya diletakkan sebelum tanda petik

77. Pernyataan yang tidak sesuai denga isi teks di atas adalah...
A. Curah hujan mulai meningkat, tetapi tinggi muka air Danau Toba masihterus menurun.
B. Level tinggi muka air Danau Toba per Selasa (6/4/2021) pun sudah berada 903,175 meter di atas permukaan laut.
C. PLTA terancam berhenti beroperasi.
D. Teknologi modifikasi cuaca dilakukan dengan menyemprotkan bahan-bahan kimia ke awan.
E. Teknologi modifikasi cuaca diterapkan untuk menahan penurunan TMA Danau Toba.

Jawaban: D
Teknologi modifikasi cuaca dilakukan dengan menyemprotkan bahan-bahan kimia > seharusnya yang benar adalah menyemprotkan bahan semai flare cosat berupa kalsium klorida dan magnesium klorida......

78. Proses afiks yang tidak sesuai dengan kaidah kebahasaan ditemukan pada ...
A. Kalimat 2
B. Kalimat 4
C. Kalimat 5
D. Kalimat 6
E. Kalimat 7

Jawaban: D
Proses imbuhan yang tidak sesuai kaidah kebahasaan terdapat pada kalimat 6 > mensemprotkan seharusnya menyemprotkan. Kata berawal (k,t,s,p) luluh apabila ditambahkan dengan imbuhan me/pe

Teks ini digunakan untuk menjawab soal nomor 79-81.
(1) Baru-baru ini, sejumlah ilmuwan menemukan spesies tokek teranyar yang diberi nama jackie's day (Cnemaspis jackieii). (2) Nama ini terinspirasi dari tokoh bela diri yang sekaligus aktor tenar, Jackie Chan. (3) Penamaan ini disebabkan oleh kelincahan si tokek yang membuat para ilmuwan teringat akan sosok Jackie Chan yang gesit. (4) Selain kelincahannya, ada alasan lain yang mendasari penamaan tokek tersebut. (5) Penamaan yang menarik perhatian itu akan memudahkan pelestariannya karena orang-orang akan lebih mengenal spesies tersebut.

(6) Jenis tokek ini pertama kali ditemukan oleh herpetolog di Pegunungan Ghats Barat, India. (7) Dari ciri-ciri fisiknya, Cnemaspis jackieii memiliki tubuh yang cukup kecil dengan ukuran 3-4 sentimeter. (8) Sebagai makhluk diurnal, tokek jenis ini mempunyai sepasang mata dengan pupil sirkular. (9) Hal ini berbeda dengan tokek nokturnal yang memiliki pupil irisan.

(10) Saat penemuan itu, para ilmuwan tidak hanya menemukan spesies Cnemaspis jackieii, tetapi juga menemukan sebelas spesies lainnya. (11) Jenis-jenis tersebut juga memiliki nama yang tak kalah unik. (12) Contohnya adalah Cnemaspis balerion yang diambil dari laga "Game of Thrones", Cnemaspis smaug yang terinspirasi dari laga "Lord of the Rings", dan Cnemaspis galaxia yang tubuhnya mirip dengan galaksi dan bintang-bintang.

79. Paragraf 2 bacaan tersebut berfungsi untuk ....
A. menggambarkan spesies tokek Cnemaspis jackieii
B. mendeskripsikan penemuan spesies tokek terbaru
C. menegaskan adanya penemuan spesies tokek jackie's day
D. menunjukkan bukti tentang penemuan spesies tokek terbaru
E. memberikan pemahaman baru tentang penemuan spesies jackie's day

Jawaban: A 
Pembahasan:
Sebuah teks yang padu memiliki hubungan antara paragraf satu dengan paragraf lainnya yang terjalin secara logis. Hubungan antarparagraf dapat berupa perincian, contoh, pemaparan, penjelasan, akibat, penambahan, simpulan, atau pertentangan. Untuk mengetahui hubungan setiap paragrafnya, pembaca perlu mengetahui gagasan pokok yang dibahas pada setiap paragrafnya.
Paragraf pertama teks tersebut menginformasikan bahwa ada penemuan spesies tokek terbaru.
Paragraf kedua teks tersebut mendeskripsikan spesies tokek terbaru dengan menyebutkan lokasi penemuannya dan ciri-ciri fisiknya.
Paragraf ketiga teks tersebut menjelaskan bahwa selain spesies Cnemaspis jackieii, para ilmuwan menemukan sebelas spesies tokek lainnya yang diberi nama unik.
Berdasarkan penjelasan tersebut, paragraf 2 teks tersebut berfungsi untuk menggambarkan spesies tokek terbaru, yaitu Cnemaspis jackieii.

80. Frasa jenis-jenis tersebut pada kalimat (11) merujuk pada ....
A. sebelas tokek
B. spesies lainnya
C. Cnemaspis jackieii
D. spesies tokek terbaru
E. tokek dengan nama unik

Jawaban: B 
Pembahasan:
Salah satu syarat sebuah paragraf dapat dikatakan padu adalah adanya kohesi. Kohesi menjadi penanda bahwa kalimat-kalimat dalam sebuah paragraf memiliki keterkaitan. Kohesi dapat berupa kata ganti orang, kata tunjuk, konjungsi antarkalimat, repetisi, atau sinonim. Pada kalimat (11) terdapat frasa jenis-jenis tersebut yang mengandung sebuah kohesi, yaitu kata tersebut. Karena terdapat kohesi, kalimat (11) merujuk pada kalimat sebelumnya. Frasa jenis-jenis tersebut pada kalimat (11) merujuk pada sebelas spesies tokek lainnya yang dijelaskan pada kalimat (10). Berdasarkan penjelasan tersebut, kalimat (11) menjelaskan bahwa sebelas spesies tokek lainnya juga memiliki nama yang unik.

81. Gagasan pada kalimat (5) dapat disampaikan melalui kalimat ...
A. Karena penamaan yang menarik perhatian itu, orang-orang akan lebih mengenal spesies tersebut dan akan memudahkan pelestariannya.
B. Melalui penamaan yang menarik perhatian itu, spesies tersebut menjadi lebih mudah dilestarikan dan dikenali oleh orang-orang.
C. Penamaan yang menarik perhatian itu membuat orang-orang akan lebih mengenal spesies tersebut karena akan memudahkan pelestariannya.
D. Pelestarian spesies tersebut menjadi lebih mudah karena penamaan yang menarik perhatian itu membuat orang-orang akan lebih mengenalnya.
E. Orang-orang akan mengenal spesies tersebut dan akan lebih mudah untuk melestarikannya sebab penamaan yang menarik perhatian itu.

Jawaban: D 
Pembahasan:
Gagasan adalah informasi yang disampaikan, baik dalam kalimat maupun dalam paragraf. Gagasan pada kalimat dapat ditentukan berdasarkan struktur intinya, yaitu subjek dan predikat. Berikut analisis pola kalimat (6).
Penamaan yang menarik →  S 
perhatian itu akan memudahkan →  P 
pelestariannya →  O
karena →  Konj.
orang-orang →  S
akan lebih mengenal →  P 
spesies tersebut. →  O
Kalimat (6) memiliki tiga informasi yang saling berkaitan, yaitu
1. penamaan yang menarik perhatian
2. pelestarian spesies tersebut menjadi mudah
3. orang-orang akan lebih mengenal spesies tersebut
Dari informasi tersebut, poin 1 menyebabkan poin 2 (penamaan yang menarik perhatian menyebabkan pelestarian spesies menjadi mudah), sedangkan poin 2 terjadi karena poin 3 (pelestarian spesies yang menjadi mudah terjadi karena orang-orang-orang lebih mengenal spesies tersebut).
Berdasarkan penjelasan tersebut, gagasan pada kalimat (6) dapat diungkapkan melalui kalimat yang ada di pilihan jawaban D.

Teks ini digunakan untuk menjawab soal nomor 82 dan 83.
(1) Infertilitas adalah masalah yang ditakuti oleh pasangan suami istri yang berharap mendapatkan momongan. (2) Banyak faktor yang dapat menyebabkan ketidaksuburan, salah satunya adalah obat-obatan yang rutin dikonsumsi. (3) Pada perempuan, siklus menstruasi dikontrol secara ketat oleh interaksi antara otak, ovarium, dan rahim. (4) Masalah kesehatan atau obat apa pun yang mengganggu komunikasi ini dapat berdampak buruk pada ovulasi dan membuat perempuan sulit untuk 
hamil.

(5) NSAID atau non-steroidal anti-inflammatory drugs adalah obat yang digunakan untuk meredakan nyeri. (6) Beberapa obat ini bisa didapatkan secara bebas tanpa resep dokter. (7) Perempuan muda yang secara rutin mengonsumsi NSAID dapat mengalami penurunan kesuburan. (8) Dijelaskan dalam laman WebMD, obat obatan NSAID dapat menghambat ovulasi dan menurunkan kadar hormon progesteron perempuan. (9) Efek ini sebenarnya tidak permanen, tetapi seorang perempuan tidak akan hamil jika dia terus menggunakan NSAID. (10) Oleh karena itu, perempuan yang berencana hamil disarankan untuk berhenti minum obat-obatan NSAID.

82. Kalimat yang dapat melemahkan argumen pada paragraf pertama adalah ...
A. Masa ovulasi sebaiknya digunakan sebaik mungkin agar peluang hamil menjadi lebih besar.
B. iklus menstruasi pada setiap perempuan berbeda-beda bergantung pada teratur atau tidaknya siklus tersebut.
C. Tidak menutup kemungkinan konsumsi obat-obatan juga dapat membantu untuk memperbesar peluang kehamilan.
D. Setiap obat memiliki karakteristiknya masing-masing sehingga harus diteliti sebelum dikonsumsi.
E. Stres berlebihan pun dapat mengganggu komunikasi antara otak, ovarium, dan rahim.

Jawaban: C 
Pembahasan:
Kalimat yang dapat melemahkan argumen adalah kalimat yang membantah pernyataan pada sebuah paragraf. Untuk menjawab soal seperti ini, kita perlu mengetahui hal apa yang dibicarakan pada paragraf yang hendak dilemahkan. Dalam soal ini, paragraf yang ditanyakan adalah paragraf pertama.
Ide pokok paragraf pertama adalah salah satu faktor yang dapat menyebabkan infertilitas pada perempuan adalah konsumsi obat-obatan. Pernyataan ini dapat dilemahkan oleh kalimat pada jawaban C karena jawaban C membicarakan obat-obatan, tetapi dari sisi yang lebih positif terhadap perempuan, yakni obat-obatan yang dapat membantu memperbesar peluang kehamilan.

83. Khasiat susu bagi tubuh kita sudah tidak diragukan lagi. Meskipun 
demikian, tidak setiap orang bersedia mengonsumsi susu. Ada dua  kemungkinan penyebabnya: pertama, karena sifat yang terkandung dalam susu yang tidak disukai orang; kedua, karena sifat biologis orang yang bersangkutan (intoleran), yang ditandai dengan gangguan pencernaan seperti diare, perut kembung, dan sering buang angin setelah minum susu.

Penyebab pertama dapat diatasi dengan penambahan sari jeruk, markisa, apel, atau lainnya sehingga rasa asli susu yang memualkan dapat dihilangkan. Sementara itu, penyebab kedua dapat diatasi dengan menggantinya dengan air susu yang telah mengalami perlakuan khusus, yaitu fermentasi. Secara biologis, penderita intoleran susu tidak mampu mencerna laktosa dari makanan atau minuman dalam susu sehingga terjadi penimbunan laktosa dalam usus. Penderita yang demikian dapat meminum susu bubuk dengan kadar laktosa rendah atau air susu fermentasi, seperti yoghurt, kefir, dan koumis.

Berdasarkan bacaan di atas, bila seorang temanmu, Ari, mengeluh bahwa ia sebenarnya ingin minum susu seperti teman lainnya, tetapi selalu diare ketika minum susu. Apa yang akan kamu sarankan?
A. Ari dapat mencoba minum susu dengan menambah sari jeruk.
B. Ari dapat mencoba susu bubuk seperti susu yang diminum oleh balita.
C. Ari dapat mencoba minum yoghurt.
D. Ari tidak perlu minum susu sama sekali.
E. Ari dapat mencoba minum susu secara bertahap

Jawaban: C 
Pembahasan:
Kalau ada soal literasi Bahasa Indonesia yang teksnya panjang, kamu bisa lihat soalnya dulu baru teksnya untuk menghemat waktu. Di soal dikatakan bahwa Ari ingin minum susu juga seperti teman lainnya tapi selalu diare saat minum susu. Di teks bisa kita temukan dua penyebab diare. Nah, kondisi Ari lebih tepat untuk penyebab kedua yaitu gangguan pencernaan. Lanjut ke akhir teks, kamu bisa mengetahui bahwa untuk mengatasi gangguan pencernaan adalah dengan mengganti air susu dengan yoghurt, kefir, atau koumis. Berarti jawaban yang paling tepat adalah C.

The questions number 84-86 are based on the following passage. 
Loneliness and social isolation in older adults are serious public health risks affecting a significant number of people in the United States and putting them at risk for dementia and other serious medical conditions. A report from the National Academies of Sciences, Engineering, and Medicine (NASEM) points out that more than one-third of adults aged 45 and older feel lonely, and nearly one-fourth of adults aged 65 and older are considered to be socially isolated. Older adults are at increased risk for loneliness and social isolation because they are more likely to face factors such as living alone, the loss of family or friends, chronic illness, and hearing loss.

Although it's hard to measure social isolation and loneliness precisely, there is strong evidence that many adults aged 50 and older are socially isolated or lonely in ways that put their health at risk. Recent studies found that social isolation significantly increased a person's risk of premature death from all causes, a risk that may rival those of smoking, obesity, and physical inactivity. Poor social relationships (characterized by social isolation or loneliness) was also associated with a 29% increased risk of heart disease. Loneliness among heart failure patients was associated with a nearly 4 times increased risk of death, 68% increased risk of hospitalization, and 57% increased risk of emergency department visits.

84. The writer's intention in writing the passage is to ....
A. disseminate the information about the impacts loneliness and social isolation in the elderly
B. encourage people to be more concerned about the elderly who are lonely and socially isolated
C. prove that the elderly experiencing loneliness and social isolation actually exist in the United States
D. convey the pain and sadness experienced by lonely and socially isolated elderly based on research
E. tell about the loneliness and unhappiness felt by the elderly who are socially isolated, as well as research findings

Jawaban: A 
Pembahasan:
Soal menanyakan tujuan si penulis menulis teks. Paragraf pertama menyampaikan bahwa kesepian dan isolasi sosial pada lansia adalah risiko kesehatan masyarakat yang serius yang memengaruhi sejumlah besar orang di Amerika Serikat. Paragraf kedua menyampaikan hasil penelitian tentang dampak kesepian dan isolasi sosial yang dialami oleh para lansia.
Secara umum, teksnya membahas dampak buruk dari rasa kesepian dan isolasi sosial bagi mereka yang berusia tua. Dengan demikian, tujuan teks tersebut adalah menyebarluaskan informasi tentang dampak rasa kesepian dan isolasi sosial pada lansia sehingga pilihan A tepat.

85. To whom is the text likely addressed?
A. Families staying with elderly
B. People living around elderly
C. Pensioners who have just left their jobs
D. Senior citizens living alone in the suburbs
E. Employers having old workers in their companies

Jawaban: B 
Pembahasan:
Soal menanyakan sasaran pembaca teks. Secara umum, teks bacaan membahas tentang rasa kesepian dan isolasi sosial yang dialami para lansia yang berdampak buruk pada kesehatan mereka. Dengan demikian, sasaran teks ini adalah mereka yang tinggal di sekitar lansia agar mereka memperhatikan kondisi para lansia tersebut. Jadi, pilihan B tepat.

86. From the last paragraph, we can infer that ...
A. Those who experience loneliness may develop heart disease.
B. Feeling lonely aggravates the heart condition of the elderly.
C. Heart disease is the worst ailment that a lonely older person will suffer from.
D. The older people get, the more lonely they will feel.
E. Loneliness and social isolation that people experience makes them unable to survive.

Jawaban: B 
Pembahasan:
Soal menanyakan kesimpulan tersirat yang dapat diambil berdasarkan informasi pada teks. Berdasarkan kalimat ketiga dan keempat, dapat diambil kesimpulan bahwa rasa kesepian akan memperburuk kondisi jantung lansia. Maka dari itu piliha B paling tepat.

The questions number 87-88 are based on the following passage. 
Those in the retail industry that are successful in digitally transforming will be ones that leverage technology to innovate around an immersive and disruptive customer experience. These "lighthouse" retailers are the ones driving innovation, adopting emerging technologies early, and showing the industry what works and is possible. They are the ones experimenting with new models and store formats, playing with recombinations/reconfigurations of scale, assets, product, place, and people. The "store of the future" is the keystone of this effort.

The store of the future will combine multiple formats to deliver an immersive and unique experience across all customer touchpoints online and in the store. It will be heavily instrumented for operational efficiency and digitally connected end-to-end to leverage a single view of customer, product, and order. The store of the future - despite its reliance on sophisticated technology and infrastructure - will be trying to look smaller and less intimidating, more personalized, and frictionless.

The deployment of these and other digital technologies in disruptive ways can turn visitors into shoppers and shoppers into brand-loyal customers. However, understanding which consumers want personalization and which want contextualized and relevant experiences, while protecting everyone's privacy, has grown more difficult. With so much at stake, data privacy has become a challenge for every party involved.

87. Who needs to see the passage?
A. Online customer
B. Businessperson
C. Manufacturer
D. Importer
E. Distributor
 

Jawaban: B
Pembahasan:
Soal tersebut menanyakan sasaran pembaca yang tepat untuk teks ini. 
Untuk mengetahuinya, perlu dipahami ide pokok dari teks terlebih dahulu.
Pada paragraf 1, penulis mendeskripsikan kualitas retailer yang sukses 
melakukan transformasi digital.
Pada paragraf 2, penulis menggambarkan format toko di masa depan. 
Pada paragraf 3, penulis menjelaskan dampak negatif dari penggunaan teknologi bagi pelanggan, yaitu terkait dengan keamanan data mereka. 
Oleh karena itu, yang paling mungkin menjadi sasaran teks adalah orang yang berwenang atau yang dapat membuat kebijakan untuk mengubah format ritel menjadi digital dan mempelajari langkah-langkah yang harus diterapkan supaya sukses dalam transformasinya. Dari opsi jawaban, sasaran pembaca yang paling tepat adalah businessperson, yaitu orang yang bekerja di bidang bisnis, terutama orang yang memiliki posisi penting di perusahaan. Dengan demikian, pilihan B tepat.

88. Measles, a childhood disease, has caused suffering to mankind for thousands of years. However, the search for an effective measles vaccine lasted two hundred years and has finally ended in success. Now, for the first time, measles is a preventable disease. You may ask, "How is this important to children?" Every year measles kills twice as many Americans as polio does. More children die from measles than from any other common childhood disease. Also complications of some degree occur in about one child out of six. Most complication include pneumonia and ear disorders. Another after-effect of measles-brain damage is less common, but it can have such serious consequence that it deserves special attention. Brain damage due to measles sounds like something far away from our experience. In reality, it is not. Like other injury, damage to the brain can be very slight or very severe. It is quite possible that we have never seen or heard a child who has severe brain damage - the child would either have died or would be in an institution. However, in medical research a relation has been found between measles and such things as behavior problems, personality changes and dulling of metal ability. For example, a child may be bad-tempered or a little slow to learn after he has recovered from measles.

One of the important findings of the research on measles is that ....
A. children who have got measles may become difficult to handle because of their behavior.
B. in reality, there are no measles patients who get brain damage.
C. personality changes already occur at the time a child has measles.
D. measles can cause children to become physically handicapped.
E. measles is the first killer of childhood disease in the world

Jawaban: A 
Pembahasan:
Untuk mengerjakan contoh soal skolastik UTBK literasi Bahasa Inggris ini, kamu perlu mengetahui kalimat kunci yang menjadi inti dari teks. Dalam dua kalimat terakhir, dijelaskan bahwa ada kaitan antara measles dengan perubahan sikap, kepribadian, dan penumpulan kemampuan mental, misalnya setelah menderita measles, anak jadi sering marah-marah atau lamban dalam belajar. Perilaku seperti ini memungkinkan seorang anak menjadi sulit ditangani setelah menderita measles. Maka, jawaban yang tepat adalah A.

89. Which of the following statements is correct?
A. Customers demanded stores with cutting-edge infrastructure to be user-friendly.
B. The more advanced the technology is, the more successful the retailers will be.
C. Each customer will receive a better and more unique experience in online shopping.
D. Providing a one-of-a-kind shopping experience will require a combination of formats.
E. Stores of the future are trying to achieve the ultimate goal that is to attract more visitors.

Jawaban: D 
Pembahasan:
Soal menanyakan pernyataan yang tepat berdasarkan teks. Pilihan A tidak tepat. Informasi tentang toko yang menggunakan infrastruktur mutakhir ada di paragraf dua kalimat tiga. Disebutkan bahwa meskipun bergantung dengan teknologi dan infrastruktur yang canggih, toko-toko ini akan berusaha untuk bisa digunakan dengan lebih mudah (oleh pelanggan), merujuk pada kata less intimidating dan frictionless. Namun, tidak ada informasi bahwa hal ini adalah tuntutan dari pelanggan.
Pilihan B tidak tepat. Kesuksesan perusahaan ritel disebutkan di paragraf satu kalimat satu. Disampaikan bahwa mereka yang sukses adalah yang memanfaatkan teknologi untuk berinovasi. Kemutakhiran dari teknologi itu sendiri tidak menentukan kesuksesan mereka, melainkan pemanfaatannya.
Pilihan C tidak tepat. Memberikan pengalaman berbelanja yang unik disampaikan di paragraf dua kalimat satu. Tertulis bahwa pengalaman berbelanja tersebut ditujukan pada semua titik, tidak hanya di toko online namun juga di toko offline.
Pilihan D tepat. Informasi ini juga terdapat di paragraf dua kalimat satu. Disebutkan bahwa toko di masa depan memang akan menggabungkan berbagai format untuk memberikan pengalaman berbelanja yang berbeda. Dengan demikian, pernyataan informasi ini sesuai.
Pilihan E tidak tepat. Informasi yang berhubungan dengan pengunjung ada di paragraf tiga kalimat satu. Disampaikan bahwa pebisnis ritel mengerti bahwa penerapan teknologi ini bisa mengubah pengunjung menjadi pembeli, pembeli menjadi pelanggan setia. Tidak disebutkan kalau menarik lebih banyak pengunjung adalah tujuan akhir mereka.

The questions number 90-99 are based on the following passage. 

In the following article, the author speculates about a connection between the low-fat, high-carbohydrate diet recommended by the medical establishment in the last twenty years and the increasing rate of obesity among Americans.

American dietitians and members of the medical community have ridiculed low-carbohydrate diets as quackery for the past thirty years, while extolling a diet that cuts down on fat, limits meat consumption, and relies on carbohydrates as its staple. Many Americans are familiar with the food pyramid promoted by the U.S. government, with its foundation of carbohydrates such as breads, rice, and pasta, and its apex allotted to fats, oils, and sweets. Adhering to the government's anti-fat, pro-carbohydrate gospel, food manufacturers have pumped out fat-free grain products that lure consumers with the promise of leaner days. Then, why are Americans getting so fat? Could the dietary recommendations of the last twenty years be wrong? And what's more, could the proponents of diets that push protein and fat be right?

Fact: Obesity rates have soared throughout the country since the 1980s. The United States Centers of Disease Control reports that the number of obese adults has doubled in the last twenty years. The num-ber of obese children and teenagers has almost tripled, increasing 120% among African-American and Latino children and 50% among white children. The risk for Type 2 diabetes, which is associated with obesity, has increased dramatically as well. Disturbingly, the disease now affects 25% to 30% of children, compared with 3% to 5% two decades ago.

What is behind this trend? Supersized portions, cheap fast food, and soft drinks combined with a sedentary lifestyle of TV watching or Internet surfing have most likely contributed to the rapid rise of obesity. Yet, there might be more to it: is it a coincidence that obesity rates increased in the last twenty years-the same time period in which the low-fat dietary doctrine has reigned? Before the 1980s, the conventional wisdom was that fat and protein created a feeling of satiation, so that overeating would be less likely. Carbohydrates, on the other hand, were regarded as a recipe for stoutness. This perception began to change after World War II when coronary heart disease reached near epidemic proportions among middle-aged men. A theory that dietary fat might increase cholesterol levels and, in turn, increase the risk of heart disease emerged in the 1950s and gained increasing acceptance by the late 1970s. In 1979, the focus of the food guidelines promoted by the United States Depart-ment of Agriculture (USDA) began to shift away from getting enough nutrients to avoiding excess fat, saturated fat, cholesterol, and sodium-the components believed to be linked to heart disease. The anti-fat credo was born.

To date, the studies that have tried to link dietary fat to increased risk of coronary heart disease have remained ambiguous. Studies have shown that cholesterol-lowering drugs help reduce the risk of heart disease, but whether a diet low in cholesterol can do the same is still questionable. While nutrition experts are debating whether a low-fat, carbohydrate-based diet is the healthiest diet for Americans, nearly all agree that the anti-fat message of the last twenty years has been over-simplified. For example, some fats and oils like those found in olive oil and nuts are beneficial to the heart and may deserve a larger propor-tion in the American diet than their place at the tip of the food pyra-mid indicates. Likewise, some carbohydrates that form the basis of the food pyramid, like the "refined" carbohydrates contained in white bread, pasta, and white rice, are metabolized in the body much the same way sweets are. According to one Harvard Medical School researcher, a breakfast of a bagel with low-fat cream cheese is "meta-bolically indistinguishable from a bowl of sugar."

So what about those high-fat, protein diets that restrict carbohy-drates like the popular Atkins' diet and others? A small group of nutrition experts within the medical establishment find it hard to ignore the anecdotal evidence that many lose weight successfully on these diets. They are arguing that those diets should not be dismissed out of hand, but researched and tested more closely. Still others fear that Ameri-cans, hungry to find a weight-loss regimen, may embrace a diet that has no long-term datal about whether it works or is safe. What is clear is that Americans are awaiting answers and in the meantime, we need to eat something.

90. The passage is primarily concerned with
A. questioning the dietary advice of the past two decades.
B. contrasting theories of good nutrition.
C. displaying the variety of ways one can interpret scientific evidence.
D. debunking the value of diets that restrict carbohydrates.
E. isolating the cause of the rising rate of obesity.

Jawaban: A
Pembahasan: The article raises the question, Could the dietary recommendations of the last twenty years be wrong? (lines 10-11). 

91. The author's attitude toward the medical experts who ridiculed low-carbohydrate diets as quackery and praised low-fat diets is one of
A. bemused agreement.
B. seeming ambivalence.
C. unconcerned apathy.
D. implicit objection.
E. shocked disbelief.

Jawaban: D
Pembahasan: The author expresses her objection by depicting the medical experts as extreme, ridicul[ing] (line 2) one diet while extolling (line 3) another.

92. The term gospel as it is used in the passage most nearly means
A. one of the first four New Testament books.
B. a proven principle.
C. a message accepted as truth.
D. American evangelical music.
E. a singular interpretation.

Jawaban: C
Pembahasan
Pembahasan: Choices a and d are alternate definitions that do not apply to the passage. The author uses gospel (line 8) with its religious implications as an ironic statement, implying that scientists accepted a premise based on faith instead of on evidence.

93. The author uses the word Fact in order to
A. draw a conclusion about the USDA'S dietary recommendations.
B. imply that statistical information can be misleading.
C. hypothesize about the health effects of high-fat, protein diets.
D. introduce a theory about the increased rate of obesity.
E. emphasize a statistical reality regardless of its cause.

Jawaban: E
Pembahasan: The author begins with Fact (line 14) to introduce and 
highlight statistical information. She or he does not speculate about the 
meaning of the statistics until the next paragraph.

94. The passage suggests that the obesity trend in the United States is
A. partly a result of inactive lifestyles.
B. the predictable outcome of cutting down on saturated fat.
C. a cyclical event that happens every twenty years.
D. unrelated to a rise in diabetes cases.
E. the unfortunate byproduct of the effort to reduce heart disease.

Jawaban: A
Pembahasan: The author names a sedentary lifestyle of TV watching and Internet surfing (lines 24-25) as a contributing factor to the rise in obesity rates.

95. In lines 26-31, the author implies that the government's 1979 food 
guidelines
A. relied more on folk wisdom than on scientific study.
B. was based on the theoretical premise that eating less dietary fat reduces heart disease.
C. was negligent in not responding to the increasing incidence of heart disease.
D. no longer bothered to mention nutrient objectives.
E. was successful in reducing heart disease rates.

Jawaban: B
Pembahasan: The passage suggests that the 1979 dietary guidelines responded to a theory that dietary fat (line 34) increases heart disease.

96. The author characterizes the anti-fat message of the last twenty years as
A. elusive.
B. questionable.
C. incoherent.
D. beneficial.
E. inventive.

Jawaban: B
Pembahasan: The passage describes the anti-fat message as oversimplified (lines 48-49) and goes on to cite the importance of certain beneficial types of fat found in olive oil and nuts (lines 38-39).

97. The author cites the example of a breakfast of a bagel with low-fat cream cheese in order to
A. show that getting a nutritional breakfast can be fast and convenient.
B. demonstrate that carbohydrates are the ideal nutrient.
C. overturn the notion that a carbohydrate-based breakfast is necessarily healthy.
D. persuade readers that they should eat eggs and sausage for breakfast.
E. argue that Americans should greatly restrict their carbohydrate intake.

Jawaban: C
Pembahasan: This example supports the claim that the body uses refined carbo-hydrates in much the same way (lines 42-43) that it does sweets.

98. The author of the passage would most likely agree with which 
statement?
A. The federal government knowingly gave the public misleading advice.
B. Soaring obesity rates are most certainly a result of low-fat diets.
C. Nutritionists should promote high-fat, protein diets like the Atkin's diet.
D. Scientists should investigate every fad diet with equal scrutiny.
E. There is no definitive evidence connecting dietary fat to heart disease.

Jawaban: E
Pembahasan: Lines 42-43 support this statement.

99. The tone of the last sentence of the passage is one of
A. optimism.
B. resolve.
C. indulgece.
D. irony.
E. revulsion.

Jawaban: D
Pembahasan: The last sentence is ironic-it expresses an incongruity between conflicting dietary advice that targets different types of food as unhealthy, and the reality that humans need to eat.

100. Read the text and answer the question!
Earthquake is any sudden shaking of the ground caused by the passage of seismic waves through Earth's rocks. Seismic waves are produced when some form of energy stored in Earth's crust is suddenly released, usually when masses of rock straining against one another suddenly fracture and "slip." Earthquakes occur most often along geologic faults, narrow zones where rock masses move in relation to one another. The major fault lines of the world are located at the fringes of the huge tectonic plates that make up Earth's crust. 

Little was understood about earthquakes until the emergence of seismology at the beginning of the 20th century. Seismology, which involves the scientific study of all aspects of earthquakes, has yielded answers to such long-standing questions as why and how earthquakes occur. About 50,000 earthquakes large enough to be noticed without the aid of instruments occur annually over the entire Earth. Of these, approximately 100 are of sufficient size to produce substantial damage if their centers are near areas of habitation. Very great earthquakes occur on average about once per year. Over the centuries they have been responsible for millions of deaths and an incalculable amount of damage to property.

What will happen when seismic waves pass through Earth's rocks?
A. It will answer why and how earthquakes occur.
B. It may cause an earthquake.
C. It will store some form of energy in Earth's crust.
D. The masses of rock will slip.
E. It may produce seismic waves.

Jawaban: B 
Pembahasan:
Soal tersebut menanyakan hal yang akan terjadi ketika gelombang seismik melewati batuan bumi.
Informasi tentang gelombang seismik dapat ditemukan dalam paragraf pertama. Dikatakan bahwa gempa bumi 'adalah setiap guncangan tiba-tiba dari tanah yang disebabkan oleh lewatnya gelombang seismik melalui batuan bumi.' Berdasarkan pendefinisian tersebut, dapat disimpulkan bahwa jika lempengan bumi menerima gelombang seismik, gempa bumi akan terjadi.
Jadi, pilihan jawaban yang tepat adalah B.

101. Lima sekawan Sano, Joko, Adi, Rimba, dan Ratu selalu semangat berangkat bersama menuju sekolah. Joko selalu menjemput Sano, setelah ia dijemput oleh Adi. Rimba menjadi anak terakhir yang dijemput. Sementara rumah Ratu terletak di antara rumah Joko dan rumah Adi. Berikut ini pernyataan yang BENAR adalah …

A. Rumah Ratu terletak paling jauh
B. Rumah Adi terletak paling jauh
C. Rumah Rimba terletak paling jauh
D. Rumah Sano terletak paling dekat
E. Rumah Adi terletak paling dekat

Jawaban: B
Pembahasan:
Untuk menjawabnya, kita harus tahu siapa yang berangkat lebih dulu ke sekolah, artinya rumah dia lebih jauh dari sekolah. Nah, dari soal tersebut, kita bisa tahu kalau rumah Adi lebih jauh dari rumah Joko, karena Joko dijemput Adi yang kemudian mereka berdua menjemput Sano. Maka, rumah Sano lebih dekat dari rumah Joko. Sementara itu, karena rumah Ratu berada di antara rumah Joko dan Adi, maka rumah Rimba lah yang paling dekat dengan sekolah, karena ia dijemput paling terakhir.
Apabila dituliskan berdasarkan urutan dari yang rumahnya terdekat dari sekolah sampai yang terjauh, maka jadinya seperti ini:
SEKOLAH – Rumah Rimba – Rumah Sano – Rumah Joko – Rumah Ratu – Rumah Adi
Jadi, jawaban yang benar sesuai dengan pilihan di atas adalah B.

102. Perhatikanlah info dibawah ini
Infografik soal utbk penalaran matematika dengan jawaban

Sampah anorganik lebih lama terurai dibandingakn dengan sampah organik. Waktu dekomposisi popok sekali pakai lebih lama dari plastik, namun kurang dari kulit sintetis. Berapa waktu dekomposisi yang mungkin dari popok sekali pakai?
A. 100 tahun
B. 250 tahun
C. 375 tahun
D. 475 tahun
E. 575 tahun

Jawaban: D
Pembahasan:
Kunci dalam mengerjakan soal ini adalah fokus ke pertanyaannya.
Pertanyaannya adalah berapa waktu dekomposisi dari popok sekali pakai? Maka, kita bisa langsung fokus ke popok sekali pakai dengan melihat informasi tambahan yang tersedia. Nah, dari informasi yang tersedia, kita bisa tahu bahwa waktu dekomposisi popok sekali pakai, lebih lama dibandingkan plastik (lebih dari 400 tahun), tetapi lebih sebentar dari kulit sintetis (kurang dari 500 tahun).
Nah, dari informasi tersebut, jawaban yang paling tepat adalah dekomposisi popok sekali pakai memakan waktu hingga 475 tahun.
Maka kunci jawaban yang tepat adalah D

Bacalah paragraf dibawah ini!
Khasiat susu bagi tubuh kita sudah tidak diragukan lagi. Meskipun demikian, tidak setiap orang bersedia mengonsumsi susu. Ada dua kemungkinan penyebabnya: pertama, karena sifat yang terkandung dalam susu yang tidak disukai orang; kedua, karena sifat biologis orang yang bersangkutan (intoleran), yang ditandai dengan gangguan pencernaan seperti diare, perut kembung, dan sering buang angin setelah minum susu. Penyebab pertama dapat diatasi dengan penambahan sari jeruk, markisa, apel, atau lainnya sehingga rasa asli susu yang memualkan dapat dihilangkan. Sementara itu, penyebab kedua dapat diatasi dengan menggantinya dengan air susu yang telah mengalami perlakuan khusus, yaitu fermentasi. Secara biologis, penderita intoleran susu tidak mampu mencerna laktosa dari makanan atau minuman dalam susu sehingga terjadi penimbunan laktosa dalam usus. Penderita yang demikian dapat meminum susu bubuk dengan kadar laktosa rendah atau air susu fermentasi, seperti yoghurt, kefir, dan koumis.

103. Berdasarkan bacaan di samping, bila seorang temanmu, Ari, mengeluh bahwa ia sebenarnya ingin minum susu seperti teman lainnya, tetapi selalu diare ketika minum susu. Apa yang akan kamu sarankan?
A. Ari dapat mencoba minum susu dengan menambah sari jeruk.
B. Ari dapat mencoba susu bubuk seperti susu yang diminum oleh balita.
C. Ari dapat mencoba minum yoghurt.
D. Ari tidak perlu minum susu sama sekali.
E. Ari dapat mencoba minum susu secara bertahap

Jawaban: C
Pembahasan:
Kalau ada soal yang teksnya panjang seperti ini, tipsnya adalah kamu sebaiknya lihat soalnya dulu, baru teksnya. Dengan begitu, kamu jadi tahu konteks pertanyaannya dan jadi menghemat waktu dalam membaca teksnya.
Nah, di soal tersebut dikatakan bahwa, Ari ingin minum susu juga seperti teman lainnya, tapi Ari selalu diare saat minum susu. Pada teks tersebut, kita bisa menemukan dua penyebab terjadinya diare.
Kalau diperhatikan, kondisi Ari lebih tepat untuk penyebab yang kedua, yaitu gangguan pencernaan. Lanjut ke teks terakhir, kamu bisa mengetahui bahwa untuk mengatasi gangguan pencernaan, bisa mengganti air susu dengan yoghurt, kefir, atau koumis.
Berarti jawaban yang paling tepat adalah C.

Read the passage below!
Measles, a childhood disease, has caused suffering to mankind for thousands of years. However, the search for an effective measles vaccine lasted two hundred years and has finally ended in success. Now, for the first time, measles is a preventable disease. You may ask, ”How is this important to children?” Every year measles kills twice as many Americans as polio does. More children die from measles than from any other common childhood disease. Also complications of some degree occur in about one child out of six. Most complication include pneumonia and ear disorders. Another after-effect of measles-brain damage is less common, but it can have such serious consequence that it deserves special attention. Brain damage due to measles sounds like something far away from our experience. In reality, it is not. Like other injury, damage to the brain can be very slight or very severe. It is quite possible that we have never seen or heard a child who has severe brain damage – the child would either have died or would be in an institution. However, in medical research a relation has been found between measles and such things as behavior problems, personality changes and dulling of metal ability. For example, a child may be bad-tempered or a little slow to learn after he has recovered from measles.

104. One of the important findings of the research on measles is that ….
A. children who have got measles may become difficult to handle because of their behavior.
B. in reality, there are no measles patients who get brain damage.
C. personality changes already occur at the time a child has measles.
D. measles can cause children to become physically handicapped.
E. measles is the first killer of childhood disease in the world

Jawaban: A
Pembahasan:
Untuk mengerjakan soal seperti ini, kamu perlu mengetahui dulu kalimat kunci yang menjadi inti dari teks. Dalam dua kalimat terakhir, dijelaskan bahwa ada kaitan antara measles dengan perubahan sikap, kepribadian, dan penumpulan kemampuan mental. Misalnya, setelah menderita measles anak jadi sering marah-marah atau lamban dalam belajar. Nah, perilaku seperti ini memungkinkan seorang anak menjadi sulit ditangani setelah menderita measles.
Maka, jawaban yang tepat adalah A.

The question is based on the following passage.
Seventeen-year-old Robin West is an anomaly among her peers, she doesn’t have a smartphone. Instead of scrolling through apps like TikTok and Instagram all day, shenuses a so-called “dumbphone”. These are basic phones, or feature phones, with very limited functionality compared to say an iPhone. You can typically only make and receive calls and SMS text messages. And, if you are lucky, you can listen to radio and take very basic photos, but definitely not connect to the internet or apps. These devices are similar to some of the first phones that people bought back in the late 1990s. Ms West’s decision to ditch her former smartphone two years ago was a spur of the moment thing. While looking for a replacement phones in a second-hand shop, she was lured by the low price of a “brick phone”. Her current phones, from French firm MobiWire, cost her just £8. And because it has no smartphone functionality, she doesn’t have an expensive monthly data bill to worry about. She didn’t notice until she bought a brick phone how much a smartphone was taking over her life. She had a lot of social media apps on it, and she didn’t get as much work done as she was always on the phone.

Dumbphones are continuing to enjoy a revival. Google searches for them jumped by 89% between 2018 and 2021, according to a report by software firm SEMrush. Fashion, nostalgia, and them appearing in TikTok videos, have a part to play in the dumbphone revival. While it’s true that dumbphones can’t compete with the latest premium Apple and Samsung models when it comes to performance or functionality, they can outshine them in equally important areas such as battery life and durability. Five years ago, Przemek Olejniczak, a psychologist, swapped his smartphone for a Nokia 3310, initially because of the longer-lasting battery. However, he soon realised that there were other benefits. Przemek Olejniczak admits that he now has to plan ahead more when he goes traveling. Before he would always be stuck to the phone, checking anything and everything, browsing Facebook or the news, or other facts he didn’t need to know. Now he has more time for his family and himself. A huge benefit is that he isn’t addicted to liking, sharing, commenting, or describing his life to other people. Now he has more privacy.

Bearne, S. (2022). Not smart but clever? The return of ‘dumbphones’. Taken on September 23, 2022 from:
https://www.bbc.com/news/business-60763168.

105. The writer says in paragraph 1 that dumbphones ….
A. are slightly different from iPhones in term of functionality
B. are the same as old phones with very limited features
C. were firstly available in stores in the late 1990s
D. lose internet connection almost all the time
E. don’t have camera or radio features

Kunci Jawaban : B
Pembahasan :
Soal menanyakan pernyataan yang benar mengenai dumbphones berdasarkan informasi pada paragraf 1.
Pernyataan yang benar adalah B (dumbphones sama dengan ponsel lama dengan fitur yang sangat terbatas) karena sesuai dengan informasi pada kalimat 3 yang menyatakan bahwa fungsi dari ponsel tersebut terbatas. Lalu, pada kalimat terakhir dikatakan bahwa perangkat ini (dumbphone) mirip dengan beberapa ponsel pertama yang dibeli orang pada akhir 1990-an. Artinya, kedua ponsel tersebut (dumbphone dan beberapa ponsel zaman dulu) sama sama memiliki fungsi yang terbatas.
Dengan demikian, pilihan jawaban yang tepat adalah B.

106. Diketahui terdapat penambahan biaya pengemasan sebesar Rp3.000,00 untuk setiap pengiriman. Jika Anggun ingin mengirim dua barang sekaligus dan satu barang terpisah, biaya termurah yang dikeluarkan Anggun adalah..
A. Rp44.650,00
B. Rp43.750,00
C. Rp37.400,00
D. Rp32.500,00
E. Rp20.150,00
 

Jawaban: D
Pembahasan:
Diketahui bahwa barang A seberat 1.450 g barang B seberat 0,5 kg = 500 g dan barang C seberat 3,2 g = 3.200 g.
Diketahui pula terdapat penambahan biaya pengemasan sebesar Rp3.000,00 untuk setiap pengiriman.
Untuk mengetahui total biaya termurah yang dikeluarkan oleh Anggun, akan diperiksa biaya yang dikeluarkan untuk setiap kemungkinan berikut.
Kemungkinan 1: Pengiriman barang A dan B digabung, sedangkan barang C terpisah.
Karena barang A digabung dengan barang B, maka beratnya menjadi 1.450g + 500g = 1.950g. Total biaya yang dikeluarkan adalah sebagai berikut.
Total biaya
= Biaya barang A dan B + biaya barang C + 2 x biaya pengemasan
= 13.250 + 13.250 + 3.000 x 2
= 26.500 + 6.000
= 32.500
Didapat bahwa biaya yang dikeluarkan Anggun pada kemungkinan 2 adalah Rp 32.500,00.
Kemungkinan 2: Pengiriman barang A dan C digabung, sedangkan barang B terpisah.
Karena barang A digabung dengan barang C, maka beratnya menjadi 1.450g + 3.200g = 4.650g. Total biaya yang dikeluarkan adalah sebagai berikut.
Total biaya
= Biaya barang A dan C + biaya barang B + 2 x biaya pengemasan
= 24.500 + 6.900 + 3.000 x 2
= 31.400 + 6.000
= 37.400
Didapat bahwa biaya yang dikeluarkan Anggun pada kemungkinan 1 adalah Rp37.400,00.
Kemungkinan 3: Pengiriman barang B dan C digabung, sedangkan barang A terpisah.
Karena barang B digabung dengan barang C, maka beratnya menjadi 500g + 3.200g = 3.700g. Total biaya yang dikeluarkan adalah sebagai berikut.
Total biaya
= Biaya barang B dan C + biaya barang A + 2 x biaya pengemasan
= 24.500 + 13.250 + 3000 x 2
= 37.750 + 6.000
= 43.750
Didapat bahwa biaya yang dikeluarkan Anggun pada kemungkinan 3 adalah Rp 43.750,00.
Berdasarkan ketiga kemungkinan tersebut, biaya termurah yang dikeluarkan Anggun adalah Rp32.500,00.
Jadi, jawaban yang benar adalah D.

107. Seorang petani ingin memelihara ayam, kucing, anjing, koloni lebah, kambing, dan beruang. Ada tiga area, yakni A, B, dan C, sebagai tempat tinggal hewan-hewan tersebut dengan ketentuan sebagai berikut.
  • Koloni lebah tidak bisa berbagi tempat dengan hewan lainnya.
  • Anjing dan kucing bisa berada di area yang sama, bisa juga tidak, tetapi harus beda area dengan beruang.
  • Kambing harus tinggal bersama ayam.
  • Beruang harus beda area dengan ayam.
Hewan yang berada sendirian di suatu area adalah ….
A. kucing
B. anjing
C. kambing
D. beruang
E. ayam

Kunci Jawaban : D
Pembahasan :
Misalkan koloni lebah berada di area A. Artinya, hewan lain hanya mungkin tinggal di area B atau C karena koloni lebah tidak bisa berbagi tempat dengan hewan lainnya. Kemudian, misalkan pula beruang berada di area B. Artinya, anjing, kucing, dan ayam tidakbisa berada di area B karena harus berbeda area dengan beruang. Tiga hewan tersebut
akan berada di area C. Lalu, kambing harus tinggal bersama ayam. Artinya, kambing berada di area C. Dengan demikian, hewan yang berada sendirian di suatu area adalah lebah dan beruang.
Jadi, jawaban yang tepat adalah D.

108. KPK  dari dua buah bilangan prima x dan y adalah 14, dengan x > y. Nilai dari 3x-y adalah…
A. -1
B. 13
C. 19
D. 21
E. 23

Kunci Jawaban: C
Pembahasan:
KPK dari 2 bilangan x dan y adalah 14.
Ketika kita jabarkan 14=2×7, maka 2 bilangan tersebut adalah  x=7 dan y=2.
Jadi, 3x – y = 37 – 2 = 21 – 2= 19
Maka, jawaban yang benar adalah C.

Teks ini digunakan untuk menjawab soal berikut.

(1) Beberapa negara kini melaporkan adanya penyusutan populasi. (2) Laporan tersebut datang dari Tiongkok dan beberapa negara lain, seperti Korea Selatan dan Jepang. (3) Setelah ditelusuri, hal itu karena warga negara, khususnya para wanita, enggan menikah serta memiliki atau membesarkan anak.

(4) Dilihat dari ranah psikologi, ada banyak faktor yang bisa membuat wanita enggan menikah dan memiliki anak. (5) Banyaknya tuntutan dan stigma pada wanita untuk menjadi ideal sesuai dengan standar sosial adalah salah satunya. (6) Itulah yang selanjutnya membuat wanita mungkin merasa terkekang karena mereka harus melakukan berbagai hal sesuai dengan tuntutan sosial. (7) Alhasil, pada era modern yang sarat akan keterbukaan informasi dan ruang berpendapat, makin banyak wanita yang justru memilih untuk tidak menikah dan mempunyai anak dengan alasan agar mereka bisa mendapatkan kebebasan yang penuh. (8) Selain faktor psikologis, makin sulitnya tantangan untuk membesarkan anak, terlebih di kota besar, juga menjadi penyebab lain yang membuat wanita enggan menikah dan memiliki anak. (9) Tantangan itu tidak hanya berpusat pada hal-hal materiel (seperti biaya) saja, tetapi juga pada masalah menjaga anak dari pergaulan bebas.

Jayanti, I S. (2022). Semakin Banyak Wanita Memilih Tidak Menikah-Punya Anak. Diambil 9 Januari 2023 dari
https://www.hariansib.com/detail/Headlines/Semakin-Banyak-Wanita-Memilih-Tidak-Menikah-Punya-Anak/all.

109. Kalimat (8) dan (9) dalam teks tersebut mengandung hubungan ….
A. contoh
B. sebab akibat
C. penjelasan
D. penambahan
E. perincian

Kunci Jawaban : C
Pembahasan :
Hubungan antarkalimat merupakan keterkaitan antara kalimat satu dengan kalimat yang lainnya yang terjalin secara logis. Kalimat (8) dalam teks tersebut membahas salah satu faktor yang membuat wanita enggan menikah dan memiliki anak. Disebutkan bahwa selain faktor psikologis, faktor yang menyebabkan hal itu terjadi adalah makin sulitnya tantangan untuk membesarkan anak. Sementara itu, kalimat (9) menjelaskan lebih lanjut terkait tantangan yang telah disebutkan pada kalimat (8). Disebutkan bahwa tantangan itu tidak hanya berpusat hal-hal materiel.
Berdasarkan penjelasan tersebut, dapat disimpulkan bahwa kalimat (9) menjelaskan lebih lanjut informasi yang terdapat dalam kalimat (8), khususnya terkait tantangan yang membuat wanita enggan menikah dan memiliki anak. Berdasarkan penjelasan tersebut, hubungan antara kalimat (8) dan (9) adalah hubungan penjelasan.
Dengan demikian, jawaban yang tepat adalah C.

Teks ini digunakan untuk menjawab soal berikut.

[…]

(1) Di sepanjang tahun 2022, terdapat banyak bahasa gaul yang muncul di media sosial. (2) Bahasa gaul ini kemudian menjadi bahasa sehari-hari para pemakai, baik di Twitter, Instagram, maupun TikTok. (3) Bahasa-bahasa gaul ini dibuat dari singkatan hingga pelesetan yang sedang ramai dibahas anak muda. (4) Itulah yang membuat banyaknya orang tua atau orang dewasa yang tidak tahu artinya, bahkan asing dengan bentuknya.

(5) Beberapa contoh bahasa gaul terbaru yang muncul adalah MLYT, bestie, dan salty. (6) MLYT mengandung arti ‘meleyot’ yang sering digunakan untuk mengekspesikan kesukaan seseorang yang berlebihan, menunjukkan bahwa dirinya lemas hingga tidak bisa berkata-kata, atau pernyataan bahwa dirinya tidak bisa berpikir lurus karena begitu menyukai sesuatu, misalnya “Duh, aku MLYT abis ngeliat Song Joong-ki.” (7) Sementara itu, bestie adalah bahasa gaul yang menggantikan panggilan teman dekat atau best friend. (8) Biasanya, bestie dipakai untuk menyapa seseorang atau sebagai panggilan sayang untuk sahabat, seperti dalam kalimat “Hai bestie, lagi ngapain bestie?” (9) Berbeda dengan MLYT dan bestie, salty sendiri dalam bahasa Inggris (…), tetapi dalam bahasa gaul anak muda, salty diartikan sebagai ‘rasa geram’ atau menunjukkan adanya hal yang membosankan atau menyebalkan, misalnya “Aku cuma mau cerita, jangan salty ya!”

110. Apa judul yang tepat untuk teks tersebut?
A. MLYT, Bestie, dan Salty sebagai Contoh Bahasa Gaul
B. Kemunculan Bahasa Gaul di Twitter, Instagram, dan TikTok
C. Bahasa Gaul dan Makna yang Terkandung di Dalamnya
D. Munculnya Bahasa Gaul Terbaru di Media Sosial
E. Terbentuknya Bahasa Gaul Terbaru di Medsos dengan Singkatan dan Pelesetan

Kunci Jawaban : D
Pembahasan :
Gagasan utama paragraf 1 adalah banyaknya bahasa gaul yang muncul di media sosial. Gagasan utama ini terkandung dalam kalimat (1) yang didukung atau diperjelas oleh kalimat (2) dan (3), sedangkan kalimat (4) hadir untuk melengkapi informasi kalimat (3). Sementara itu, gagasan utama paragraf 2 adalah uraian beberapa contoh bahasa gaul yang muncul di media sosial. Gagasan utama ini terkandung di kalimat (5) dan didukung oleh gagasan
penjelas pada kalimat (6) hingga (9).
Berdasarkan kedua gagasan utama, dapat dipahami bahwa secara keseluruhan, teks dalam soal ini membahas banyaknya bahasa gaul yang muncul di media sosial. Dapat disimpulkan bahwa judul yang mampu mewakili kesimpulan ini ada pada opsi D: Munculnya Bahasa Gaul Terbaru di Media Sosial.
Jadi, jawaban yang tepat adalah D.

Teks untuk menjawab soal nomor 111.
Istirahat yang cukup merupakan kunci tubuh yang sehat. Hindari begadang dan berbagai aktivitas yang tidak berguna di malam hari. Begadang di malam hari meningkatkan resiko terserah berbagai macam penyakit. Para ahli menyebutkan bahwa begadang bisa menyebabkan serangan jantung, darah tinggi, kanker, dan kencing manis.

111. Berdasarkan bacaan di atas, manakah pernyataan yang memperlemah pendapat para ahli tersebut?
A. Begadang di malam hari meningkatkan risiko terserang berbagai macam penyakit
B. Begadang diperlukan saat memiliki aktivitas yang berguna di malam hari
C. Begadang tidak baik bagi kesehatan tubuh
D. Orang yang sering begadang memiliki risiko terserang kanker
E. Orang yang sering begadang dapat terserang penyakit kencing manis

Jawaban: B

Teks untuk menjawab soal nomor 112.
Seseorang yang menuntut ilmu sama halnya dengan mendaki gunung. Sewaktu mendaki, ada saja rintangan seperti jalan yang licin yang membuat seseorang jatuh. Ada pula semak belukar yang sukar dilalui. Begitu juga menuntut ilmu, seseorang akan mengalami rintangan seperti kesulitan ekonomi, kesulitan memahami pelajaran, dan sebagainya. Kesimpulan yang tepat berdasarkan bacaan di atas adalah menuntut ilmu sama halnya dengan mendaki gunung yaitu banyak rintangan.

112. Manakah pernyataan berikut yang menggambarkan kualitas 
simpulan tersebut?
A. Simpulan pasti benar
B. Simpulan mungkin benar
C. Simpulan pasti salah
D. Simpulan mungkin salah
E. Simpulan tidak dapat dinilai karena informasi tidak cukup

Jawaban: B

113. Diketahui Tan x = 1/√5 dengan 180 derajat < x> Nilai dari Sin x adalah ...
A. 1/6√6
B. -1/6√6
C. 1/6√30
D. -1/6√30
E. 1/5√6

Jawaban: B

114. Ada kumpulan data terdiri 3,5,7, a.
Berapa banyak dari empat pernyataan tersebut yang bernilai benar berdasarkan informasi berikut:
1) Rata-rata kumpulan data tersebut 6 bila a=9.
2) Median kumpulan data tersebut 5 bila a=7.
3) Jangkauan kumpulan data tersebut 4 bila a=6.
4) Modus kumpulan data tersebut 3 bila a=5
A. 4
B. 0
C. 3
D. 1
E. 2

Jawaban: E
Pembahasan:
- Rata-rata adalah 6 
- Median adalah 6 
- Jangkauan adalah 4 
- Modus adalah 5

Hadiah liburan ke luar negeri akan diberikan oleh perusahaan X jika 
pekerjanya mendapat penilaian kinerja baik dua tahun berturut-turut dan 
dinominasikan oleh rekan kerja sekaligus atasannya. Pekerja Q disukai 
oleh rekan kerjanya dan mendapatkan penilaian kinerja sangat baik 
tahun kemarin.

115. Berdasarkan informasi tersebut kesimpulannya yaitu pekerja Q berhak memperoleh hadiah liburan ke luar negeri dari perusahaan X. Pernyataan mana di bawah ini yang menggambarkan kualitas kesimpulan di atas?
A. Kesimpulan tersebut pasti benar.
B. Kesimpulan tersebut mungkin benar.
C. Kesimpulan tersebut pasti salah.
D. Kesimpulan tidak relevan dengan informasi yang diberikan.
E. Kesimpulan tidak bisa dinilai karena informasi tidak cukup.

Jawaban: C 
Pembahasan:
Kesimpulan tersebut pasti salah. Karena untuk mendapatkan hadiah liburan ke luar negeri, syaratnya pekerja harus mendapatkan penilaian kinerja sangat baik dalam dua tahun berturut-turut. Sementara pekerja Q hanya mendapatkan penilaian kinerja sangat baik di tahun kemarin.

116. Ketika Dinda telah lulus SMA, ia punya pilihan untuk membuka usaha sendiri dengan penghasilan Rp 150.000,00/hari, bekerja di perusahaan orang tua dengan gaji per bulan Rp 5.500.000 atau bekerja di perusahaan teman dengan gaji Rp 6.000.000. Jika Dinda memilih membuka usaha sendiri, berapakah biaya peluang Dinda?
A. Rp 5.500.000
B. Rp 4.500.000
C. Rp 6.000.000
D. Rp 150.000
E. Rp 2.000.000 

Jawaban: A 
Pembahasan:
Biaya peluang adalah kesempatan baik yang hilang karena memilih suatu opsi. Karena Dia memilih membuka usaha sendiri, Dinda kehilangan kesempatan untuk bekerja di perusahaan orang tuanya dengan gaji Rp 5.500.000 dan bekerja pada perusahaan teman dengan gaji Rp. 6.000.000.
Kesempatan baik yang hilang adalah bekerja di perusahaan orang tuanya dengan gaji Rp 5.500.000,00, sehingga biaya kesempatan Dinda adalah Rp 5.500.000.

117. Buah mangga mengandung vitamin C dan antioksidan sehingga dapat meningkatkan kekebalan tubuh serta membantu melawan infeksi virus. Penelitian terbaru menunjukkan bahwa mengonsumsi buah mangga dapat mencegah kanker dan menurunkan kadar kolesterol.

Manakah DUA simpulan yang PALING DIDUKUNG oleh bacaan tersebut?

(i) Harga buah mangga menjadi mahal karena memiliki manfaat untuk kesehatan.

(ii) Jumlah buah mangga yang dikonsumsi memengaruhi khasiat buah mangga untuk mencegah kanker.

(iii) Banyak penderita kanker yang mencari buah mangga untuk pengobatan.

(ix) Mengonsumsi mangga disarankan agar dapat terhindar dari risiko penyakit tertentu.

(x) Banyak produk kesehatan yang menggunakan bahan dasar ekstrak  buah mangga.

Jawaban: (iii) dan (ix) 
Pembahasan:
Soal di atas menyajikan informasi tentang satu hal, lalu disajikan informasi kedua yang memperkuat informasi pertama. Peserta diminta untuk menentukan manakah alternatif jawaban yang merupakan sebuah simpulan yang PALING DIDUKUNG oleh informasi bacaan yang disajikan dalam soal.
Tipe soal di atas merupakan contoh pilihan berganda kompleks, di mana peserta harus memilih DUA simpulan yang PALING DIDUKUNG. Jika salah satu dari opsi yang dipilih ternyata bukan kunci jawaban, jawaban peserta dianggap salah.

Perhatikan bacaan berikut ini untuk menjawab soal nomor 118-124!
[1] Harga tiket pesawat yang tinggi sejak awal tahun (2019) ini membuat masyarakat sebagai konsumen riuh dan mengeluh. [2] Dalam mengapresiasi kondisi itu, pemerintah akhirnya memutuskan untuk menurunkan tarif batas atas yang berlaku mulai 15 Mei nanti, khususnya untuk pesawat bermesin jet. [3] Besaran penurunan tarif bervariasi, berkisar pada angka 12-16%. [4] Namun, harga tiket sebagian besar rute penerbangan domestik akan turun 15%. [5] Tampaknya, penurunan sebesar itu tidak berpengaruh signifikan untuk menarik minat masyarakat-harga tiket masih dianggap sangat tinggi- sehingga mereka lebih memilih moda transportasi lain, seperti bus, kereta api, atau kapal laut. [6] Pemerintah menurunkan harga tiket dengan sejumlah pertimbangan, antara lain, harga tiket yang tinggi berdampak pada inflasi dan sejumlah daerah tujuan wisata sepi wisatawan. [7] Pertimbangan lain, pemerintah berani menurunkan tarif batas atas adalah karena harga avtur yang cenderung turun, tingkat okupansi pesawat yang membaik, dan mekanisme operasional di bandara sudah baik. [8] Dengan semua kondisi itu, tingkat ketepatan waktu penerbangan juga membaik sehingga operasional maskapai lebih efisiensi.

[9] Di sisi lain, penurunan tarif batas atas ini membuat maskapai gusar karena (a) kondisi mereka berat karena margin keuntungan menipis dan (b) maskapai akan lebih terseok-seok. [10] Selain itu, penurunan ini membuat maskapai tidak bisa fleksibel karena beberapa bulan lalu pemerintah telah menaikkan tarif batas bawah dari 30% menjadi 35% dari tarif batas atas. [11] Akibatnya, maskapai tidak bisa luwes, tidak bisa memberikan harga rendah saat musim sepi penumpang dan harga tinggi saat musim ramai penumpang. [12] Padahal, fleksibilitas seperti ini sangat menolong maskapai penerbangan menjaga tingkat keterisian.

118. Judul yang tepat untuk teks tersebut adalah ....
A. Pertimbangan dalam Penurunan Harga Tiket Pesawat
B. Keluh Kesah tentang Harga Tiket Pesawat
C. Harga Tiket Pesawat Turun Lima Belas Persen
D. Dampak Harga Tiket Pesawat terhadap Inflasi
E. Perlunya Fleksibilitas dalam Penentuan Harga Tiket

Jawaban: A
Pembahasan:
Dilihat dari tema yang dibicarakan dalam teks bacaan "Pertimbangan dalam Penurunan Harga Tiket Pesawat" menjadi judul yang paling tepat.

119. Paragraf pertama teks di atas sebenarnya terdiri atas dua paragraf. 
Kalimat yang tepat mengawali paragraf kedua yaitu ....
A. Kalimat [3]
B. Kalimat [5]
C. Kalimat [4]
D. Kalimat [7]
E. Kalimat [6] 

Jawaban: E 
Pembahasan:
Isi kalimat 1-5 membicarakan tentang kebijakan penurunan tiket pesawat dan pengaruhnya terhadap minat masyarakat. Sementara, kalimat 6-8 menyebutkan macam kondisi yang membuat pemerintah mempertimbangkan untuk menurunkan harga tiket pesawat. Dengan demikian, kalimat yang paling tepat untuk mengawali paragraf kedua yaitu kalimat 6.

120. Gagasan utama yang tepat untuk paragraf selanjutnya dari teks di atas adalah ....
A. Maskapai memangkas rute yang dinilai gemuk dan kurang signifikan menyumbang pendapatan bagi maskapai
B. Wi-Fi gratis merupakan cara untuk menarik penumpang mau menggunakan jasa maskapainya
C. Maskapai harus mencari sumber pendapatan lain karena margin keuntungan tipis, seperti bagasi berbayar atau menjual makanan di pesawat
D. Pemerintah memberikan subsidi kepada maskapai yang merasa dirugikan oleh kebijakan yang dibuat pemerintah
E. Untuk maskapai yang menggunakan pesawat non jet, kerugian yang dialami tidaklah terlalu besar

Jawaban: C 
Pembahasan:
Kesimpulannya, paragraf pertama membahas tentang kebijakan penurunan tiket pesawat yang dilakukan oleh pemerintah, sedangkan paragraf kedua membicarakan dampak kebijakan tersebut terhadap maskapai. Berdasarkan hal tersebut, kita dapat mengasumsikan bahwa yang akan dibicarakan pada paragraf selanjutnya adalah solusi bagi maskapai dalam mengatasi dampak kebijakan pemerintah.
Oleh karena itu, pernyataan terkait maskapai harus mencari sumber pendapatan lain karena margin keuntungan tipis, opsi C menjadi pilihan jawaban yang paling tepat. Pasalnya, isi pernyataan tersebut memiliki hubungan paling dekat dan sesuai dengan kondisi yang dihadapi oleh maskapai yang telah dijelaskan dalam teks.

121. Perhatikan teks tadi, alasan utama pemerintah untuk menurunkan tarif batas atas harga tiket pesawat adalah ....
A. Masyarakat resah dengan tingginya harga tiket pesawat sehingga mereka lebih memilih moda transportasi lain
B. Penurunan tarif sekitar 12-16 persen tidak akan mengurangi keuntungan yang diperoleh oleh maskapai penerbangan tersebut
C. Maskapai penerbangan akan mengalami kerugian semakin besar bila harga tiket pesawat tidak dinaikkan
D. Pelayanan beberapa bandara terkemuka di tanah air sudah menggunakan teknologi terbaru, canggih, dan efisien
E. Harga tiket pesawat yang tinggi berdampak terhadap inflasi dan sejumlah destinasi wisata sepi dari kunjungan wisatawan

Jawaban: E 
Pembahasan:
Kalimat yang tepat untuk menjawab pertanyaan yaitu kalimat 6 pada teks

122. Kalimat [3] dan [4] bisa digabung dan divariasikan tanpa mengubah maknanya menjadi ....
A. Besaran penurunan tarif bervariasi dengan kisaran 12-16 persen, tetapi harga tiket sebagian besar rute penerbangan domestik akan turun 15 persen
B. Penurunan tarif batas atas sangat bervariasi pada besaran sekitar 12% dan 16%, maka harga sebagian besar rute penerbangan menjadi turun 15%
C. Besaran turunnya tarif pesawat bervariasi, berkisar 12-16 persen,  namun harga tiket sebagian besar rute penerbangan akan turun sebesar 15 persen
D. Harga tiket sebagian besar rute penerbangan domestik turun 15 persen, padahal besaran penurunan tarif bervariasi, sekitar 12-16 persen
E. Dengan besaran penurunan tarif yang bervariasi antara 12-16% harga tiket sebagian rute penerbangan diturunkan menjadi 15%

Jawaban: A 
Pembahasan:
Dilihat dari hubungan antarkalimat, kalimat pada opsi jawaban A adalah kalimat yang paling tepat apabila kalimat 3 dan 4 dalam teks tersebut digabungkan

123. Penggunaan diksi yang tidak tepat pada teks tersebut adalah ....
A. mengapresiasi (kalimat 2)
B. berkisar (kalimat 3)
C. okupansi (kalimat 7)
D. efisiensi (kalimat 8)
E. fleksibilitas (kalimat 12)

Jawaban: C 
Pembahasan:
Kata efisiensi seharusnya diganti menjadi efisien, karena lebih sesuai dengan isi yang terkandung dalam kalimat 8.

124. Kata "itu" pada kalimat 8 teks tersebut merujuk pada ....
A. Tingginya harga tiket dan berkurangnya wisatawan
B. Harga avtur turun, okupansi baik, dan operasional bandara baik
C. Tingginya inflasi dan sepinya destinasi wisata
D. Tingkat ketepatan waktu penerbangan makin membaik
E. Operasional maskapai penerbangan lebih tepat guna

Jawaban: C 
Pembahasan:
Kata "itu" pada kalimat 8 merujuk pada informasi yang disebutkan dalam kalimat sebelumnya (kalimat 7) yaitu tingginya inflasi dan sepinya destinasi wisata.

Cermati dengan saksama teks bacaan di berikut ini untuk menjawab soal nomor 125-127!

[1] Limbah medis dan sampah plastik meningkat secara drastis selama pandemi Covid-19. [2] Penggunaan masker, sarung tangan, dan alat pelindung diri (APD) sekali pakai memicu peningkatan limbah dan sampah. [3] Berdasarkan data historis, sebanyak 75% masker sekali pakai dan sampah lain terkait pandemi yang akan berakhir di tempat pembuangan sampah akhir (TPA). [4] Sementara itu, sebagian lainnya mencemari lingkungan, berserakan di jalan-jalan, selokan, dan sungai. [5] Perilaku masyarakat yang tidak bertanggung jawab memicu krisis lingkungan. [6] Program Lingkungan PBB United Nation Environment Programme (UNEP) menyebutkan bahwa limbah dan sampah ini tidak hanya memicu pada kerusakan lingkungan, tetapi juga merugikan sektor pariwisata dan perikanan dengan kerugian mencapai 40 miliar dolar. [7] Oleh karena itu, pemerintah ....

125. Kalimat (3) perlu disempurnakan dengan cara ....
A. mengganti kata berdasarkan dengan berdasar
B. mengganti kata terkait dengan berkaitan
C. menghilangkan "kata yang"
D. menghilangkan kata "akan"
E. menambah kata dari setelah 75%

Jawaban: C 
Pembahasan:
Hadirnya kata "yang" merusak struktur kalimat. Hubungan subjek dan 
predikat menjadi kacau karena hadirnya kata "yang" .

126. Kata yang harus dihilangkan pada kalimat (6) adalah ....
A. bahwa
B. hanya
C. pada
D. tetapi
E. mencapai 

Jawaban: C 
Pembahasan:
Hubungan kata "memicu" dan kerusakan tidak memerlukan kata penghubung sehingga kata penghubung pada harus dihilangkan.

127. Pernyataan yang paling tepat untuk melengkapi kalimat (7) adalah ....
A. selayaknya menanggulangi peningkatan limbah dan sampah
B. perlu menyediakan dana penangan limbah dan sampah medis
C. harus memberi sanksi kepada masyarakat yang membuang sampah
D. seharusnya menangani kerusakan lingkungan akibat sampah dan limbah
E. seharusnya membuat peraturan tentang pembuangan sampah dan limbah

Jawaban: E 
Pembahasan:
Isi keseluruhan teks adalah peningkatan sampah dan limbah karena sesuatu. Oleh karena itu, jawaban yang benar harus diarahkan pada pengaturan agar sampah tidak terus meningkat.

Cermati dengan saksama teks bacaan di berikut ini untuk menjawab soal nomor 128-130!
[1] Penelitian terbaru menyebutkan bahwa pertumbuhan ekonomi Indonesia sudah melewati titik terendah pada kuartal II 2020. [2] Namun, kini Indonesia berada pada fase pemulihan ekonomi. [3] Neraca perdagangan Indonesia yang surplus USD 8 miliar pada periode berikutnya diyakini akan mendukung ketahanan eksternal perekonomian. [4] Di pasar keuangan, kepercayaan investor juga terus meningkat. [5] Hal itu terlihat dari pergerakan Indek Harga Saham Gabungan (IHSG) dan nilai tukar rupiah dalam beberapa pekan terakhir. [6] Momentum positif perekonomian itu harus terus dijaga. [7] ... , dalam memulihkan ekonomi, upaya menjaga kesehatan masyarakat di tengah situasi pandemi Covid-19 tetap harus diutamakan.

128. Penulisan kata bercetak tebal pada teks tersebut yang salah terdapat 
pada kalimat ....
A. kuartal
B. fase
C. miliar
D. investor
E. indek

Jawaban: 
Pembahasan: E
Penulisan yang benar adalah "indeks", bukan "indek".

129. Kata sambung yang paling tepat melengkapi kalimat [7] adalah ....
A. dengan demikian
B. oleh karena itu
C. di samping itu
D. untuk itu
E. jadi 

Jawaban: B 
Pembahasan:
Kalimat 7 dan kalimat-kalimat sebelum memiliki hubungan sebab-akibat. Oleh karena itu, kata sambung yang tepat harus menunjukkan hubungan sebab-akibat, yaitu "oleh karena itu".

130. Kalimat manakah yang penulisan katanya salah?
A. Penanaman kembali daerah pasca longsor merupakan pekerjaan yang tidak mudah.
B. Kerja sama masyarakat dan pemerintah harus dijalin agar sungai terbebas dari sampah.
C. Pelestarian lingkungan masing-masing sekolah dilakukan melalui kegiatan intrasekolah.
D. Penanganan korban bencana banjir di beberapa daerah ditangani antarkementerian.
E. Pendidikan lingkungan hidup yang tidak terencana dengan baik akan kontraproduktif.

Jawaban: A 
Pembahasan:
Penulisan pasca longsor seharusnya dirangkai yaitu "pascalongsor", bukan dipisah.

Perhatikan teks dibawah ini untuk menjawab soal nomor 131-136! 
Hadiah Nobel, anugerah sains paling (131) prestisius ini, selama ini dikritik karena sedikitnya kaum perempuan penerimanya yang tahun ini, 2018, hanya ada dua perempuan penerimanya. Kedua perempuan itu adalah Donna Strickland dari Universitas Waterloo, Kanada, yang mendapat Novel Fisika dan Frances H. Arnorl dari Institut Teknologi California, Pasadena, Amerika Serikat, untuk Nobel Kimia. Situasi ini jauh lebih baik dibandingkan tahun lalu yang dari tiga kategori sains yang diumumkan, termasuk fisiologi/kedokteran, tidak (132) satupun perempuan penerimanya. (133) Sedangkan tahun ini ada perubahan, itu menunjukkan kesenjangan antara sains dan perempuan.

Sepanjang sejarah penganugerahan Nobel sejak 1901, (134) sehingga hari ketika pengumuman Nobel 2018, Rabu (3/10/2018), baru 50 perempuan pernah menerimanya atau 5 persen dari total penerima Ini pun 59 persennya (135) merupakan penerima Nobel Sastra dan Nobel Perdamaian. Di bidang sains, baru 20 perempuan pernah menerimanya, yaitu 3 orang untuk fisika, 5 orang bidang kimia, dan 12 orang bidang fisiologi/kedokteran. Satu perempuan menerima Nobel dua kali, yaitu Marie Curie untuk Nobel Fisika 1903 dan Nobel Kimia 1911. Selain soal gender, isu yang beredar di (136) komunitas sains terkait Nobel ialah warna kulit yang diidentikkan dominasi pria tua berkulit putih sebagai pemenang.

131. ....
A. prestise
B. prestesius
C. prestisius
D. "prestisius"
E. TIDAK PERLU DIPERBAIKI

Jawaban: E 
Pembahasan:
Kata "prestisius" adalah kata yang sudah tepat, sehingga tidak perlu diperbaiki.

132. ...
A. satu-pun
B. satu pun
C. "satupun"
D. satu pun
E. TIDAK PERLU DIPERBAIKI

Jawaban: B
Kata "satupun" tidak tepat penulisannya. Penulisan yang benarnya adalah satu pun, dengan "pun" yang terpisah.

133. ...
A. Namun
B. tetapi
C. walaupun
D. apalagi
E. TIDAK PERLU DIPERBAIKI

Jawaban: A 
Pembahasan
Konjungsi "sedangkan" tidak tepat pemakaiannya karena kata itu tidak bisa mengawali kalimat. Konjungsi yang benar adalah "Namun".

134. ....
A. menjelang
B. berdasarkan
C. ketika
D. hingga
E. TIDAK PERLU DIPERBAIKI

Jawaban: D 
Pembahasan:
Konjungsi yang menunjukkan hubungan sebab-akibat seharusnya diganti dengan kata "hingga" yang bisa berarti sampai.

135. ....
A. menjadi
B. adalah
C. diindikasi
D. disinyalir
E. TIDAK PERLU DIPERBAIKI

Jawaban: E 
Pembahasan:
Kata "merupakan" tidak perlu diperbaiki, karena kata itu telah sesuai dengan isi dan konteks kalimatnya.

136. .....
A. komuniti
B. "komunitas"
C. Komunitas
D. "komunitas"
E. TIDAK PERLU DIPERBAIKI

Jawaban: E 
Pembahasan:
Kata komunitas tidak perlu diperbaiki lagi, karena kata telah sesuai dengan isi dan konteks kalimatnya.

137. Bilangan yang habis dibagi 3, namun tidak habis dibagi 5 adalah...
A. 13689
B. 12345
C. 15223
D. 20579
E. 14670 

Jawaban: A 
Pembahasan:
Adapun yang termasuk ciri bilangan yang habis dibagi tiga yaitu jumlah angka pembentuknya kelipatan 3. Sementara, ciri bilangan yang habis dibagi 5 adalah angka satuannya 0 atau 5.

13689: habis dibagi 3, tetapi tidak habis dibagi 5 (benar). 
12345: habis dibagi 3 dan habis dibagi 5 (salah).
15223: tidak habis dibagi 3 dan tidak habis dibagi 5 (salah). 
20579: tidak habis dibagi 3 dan tidak habis dibagi 5 (salah). 
14670: habis dibagi 3 dan habis dibagi 5 (salah).

138. Kurva y = ax² + 2x + 1 dengan a ≠ 0 memotong sumbu-x di dua titik berbeda. Pilihlah pernyataan yang benar, berikut ini!
A. kurva terbuka ke atas
B. kurva memotong sumbu-y negatif
C. titik puncak kurva berada di kuadran I
D. kurva terbuka ke bawah
E. kurva memotong sumbu-y positif

Jawaban: E 
Pembahasan:
Jika x=0. maka y=1 sehingga kurva memotong sumbu -y positif

139. Kurva y = ax² + 2x + 1 dengan a ≠ 0 memotong sumbu-x di dua titik berbeda. Manakah pernyataan yang benar ?
A. a < 1
B. 3a > 1
C. 3a > 2
D. 6a < 1
E. a > 1 

Jawaban: A. 
Pembahasan:
Jika kurva y = ax² + 2x + 1 dengan a ≠ 0 memotong sumbu-x di dua titik berbeda, maka 2² - 4a(1) > 0, sehingga a < 1.

140. Garis dengan persamaan mana saja yang memotong garis 2x + y = 4 dan x + 2y = 2 di dua titik berbeda?
1) y = -x + 5
2) y = x - 2
3) y = 3x -1
4) y = -2x + 7
A. (1) dan (3) SAJA yang benar.
B. (2) dan (4) SAJA yang benar.
C. (1), (2), dan (3) SAJA yang benar.
D. HANYA (4) yang benar.
E. SEMUA pilihan benar.

Jawaban: A 
Pembahasan:
Gradien garis 1 tidak sama dengan gradien garis 2, sehingga garis ketiga memotong dua garis tersebut di dua titik berbeda jika: Gradiennya berbeda dengan kedua gradien garis yang lain Tidak melalui titik potong dua garis yang lain. Dengan demikian, garis dengan persamaan (1) atau persamaan (3) memotong dua garis lainnya di dua titik berbeda.

Perhatikan bacaan di bawah untuk bisa memahami dan menjawab soal nomor 141-145
1) Kemajuan teknologi tidak terlepas dari kehidupan masyarakat. 2) Berbagai informasi yang terjadi di berbagai belahan dunia kini bisa langsung kita ketahui berkat kemajuan teknologi, misalnya melalui smartphone. 3) Jika dahulu kita mengenal pepatah dunia tak selebar daun kelor, sekarang pepatah itu selayaknya berganti menjadi dunia saat ini selebar daun kelor. 4) Kecepatan akses informasi di berbagai belahan dunia membuat dunia ini seakan semakin sempit. 5) Kita dapat menyaksikan apa yang terjadi di luar negeri dari Indonesia secara langsung.

6) Perubahan yang masif pada kehidupan umat manusia dengan segala peradabannya adalah dampak kemajuan teknologi yang tidak dapat kita hindari. 7) Namun, kemajuan teknologi yang kian dahsyat ini jangan sampai menggeser jati diri kita sebagai manusia. 8) Oleh karenanya, kita harus melakukan tindakan yang bijaksana terhadap diri sendiri, keluarga, dan masyarakat luas. (Teks iadaptasi dari https://media.neliti.com/)

141. Kalimat yang tidak logis dalam bacaan di atas adalah ....
A. kalimat (1)
B. kalimat (2)
C. kalimat (5)
D. kalimat (6)
E. kalimat (7)

Jawaban: B

142. Kata bijaksana dalam kalimat (8) memiliki asosiasi dengan kata ....
A. berbudi
B. bersahaja
C. berakal
D. berperasaan
E. berpengetahuan

Jawaban: A

143. Kata yang mengandung makna bertingkat dengan kata akses dalam 
kalimat (4) adalah ....
A. lorong
B. jalan
C. jaringan
D. saluran
E. fasilitas

Jawaban: E

144. Kata pepatah pada kalimat (3) dalam bacaan di atas, memiliki makna yang sama dengan kata ....
A. ajaran
B. nasihat
C. anjuran
D. petunjuk
E. ungkapan

Jawaban: E

145. Kalimat (6) dan (7) dalam bacaan di atas mengandung hubungan ....
A. perluasan
B. penambahan
C. penegasan
D. penghubungan
E. perincian

Jawaban: B

146. Tidur dengan posisi tengkurap dianggap buruk bagi kesehatan karena dapat memberi tekanan pada bagian punggung dan leher sehingga berakibat nyeri. Selain itu, tidur tengkurap akan menyebabkan kesemutan. Walau begitu, tidur tengkurap dapat membuka saluran pernapasan sehingga mengurangi dengkuran.

Berdasarkan informasi tersebut, manakah pernyataan berikut yang PASTI BENAR?
A. Orang akan mengalami kesemutan apabila tidur dengan posisi tengkurap.
B. Kesemutan akibat tidur tengkurap mengakibatkan tidur menjadi tidak nyenyak.
C. Orang yang memiliki gangguan pernapasan bisa disembuhkan melalui tidur tengkurap
D. Nyeri punggung dan leher dialami oleh orang yang tidurnya mendengkur.
E. Dengkuran sebagian orang yang tidur tengkurap menjadi berkurang.

Jawaban:E

147. Ketika telah siap panen, jeruk akan terasa empuk saat ditekan, kulitnya berwarna kuning langsat, dan keluar aroma wangi jika tergores. Hal-hal tersebut akan membuat nilai jual jeruk semakin tinggi. Jika buah jeruk terasa keras jika ditekan, berwarna hijau, dan tidak mengeluarkan aroma, manakah kesimpulan yang BENAR?

A. Buah jeruk belum siap dipanen.
B. Buah jeruk harganya murah.
C. Buah jeruk harganya mahal.
D. Buah jeruk tidak dapat dijual.
E. Buah jeruk rasanya masam.

Jawaban: A

Perhatikan bacaan di bawah ini untuk bisa menjawab soal kemampuan bacaan dan menulis! (nomor 148 - 150)
1) Limbah medis dan sampah plastik meningkat secara drastis selama pandemi COVID-19. 2) Masker, alat pelindung diri (APD), dan sarung tangan sekali pakai mendorong peningkatan limbah dan sampah. 3) Berdasarkan data historis, ada 75% masker sekali pakai dan sampah lain terkait pandemi yang akan berakhir di tempat pembuangan sampah akhir (TPA). 4) Sementara, sebagian lainnya mencemari lingkungan, berserakan di jalan, selokan, hingga sungai. 5) Perilaku masyarakat yang tidak bertanggung jawab memicu krisis lingkungan. 6) Program Lingkungan PBB United Nation Environment Programme (UNEP) menyebutkan bahwa limbah dan sampah ini tidak hanya memicu pada kerusakan lingkungan, tetapi turut merugikan sektor pariwisata dan perikanan dengan kerugian sampai 40 miliar dolar. 7) Oleh karena itu, pemerintah ....

148. Kalimat (3) perlu disempurnakan dengan cara ....
A. mengganti kata berdasarkan dengan berdasar
B. mengganti kata terkait dengan berkaitan
C. menghilangkan kata yang
D. menghilangkan kata akan
E. menambah kata dari setelah 75%

Jawaban: C

149. Kata yang harus dihilangkan pada kalimat (6) adalah ....
A. bahwa
B. hanya
C. pada
D. tetapi
E. mencapai

Jawaban: C

150. Pernyataan yang paling tepat untuk melengkapi kalimat (7) adalah ....
A. selayaknya menanggulangi peningkatan limbah dan sampah
B. perlu menyediakan dana penangan limbah dan sampah medis
C. harus memberi sanksi kepada masyarakat yang membuang sampah
D. seharusnya menangani kerusakan lingkungan akibat sampah dan limbah
E. seharusnya membuat peraturan tentang pembuangan sampah dan limbah

Jawaban: E

Perhatikan dan pahami dengan saksama teks di bawah ini! (nomor 151 - 152)
1) Penelitian terbaru menunjukkan pertumbuhan ekonomi Indonesia sudah melewati titik terendah pada kuartal II 2020. 2) Namun, Indonesia saat ini berada pada fase pemulihan ekonomi. 3) Neraca perdagangan Indonesia yang surplus USD 8 miliar pada periode berikutnya diyakini dapat mendukung ketahanan eksternal perekonomian. 4) Di pasar keuangan, kepercayaan investor juga kian meningkat. 5) Hal ini tampak dari pergerakan Indek Harga Saham Gabungan (IHSG) dan nilai tukar rupiah dalam beberapa pekan terakhir. 6) Momentum positif perekonomian itu harus terus dijaga. 7) [...], dalam memulihkan ekonomi, upaya menjaga kesehatan masyarakat di tengah situasi pandemi COVID-19 tetap harus diutamakan.

151. Penulisan kata bercetak tebal pada teks tersebut yang salah terdapat 
pada kalimat..
A. kuartal
B. fase
C. miliar
D. investor
E. indek

Jawaban: E

152. Kata sambung yang paling tepat melengkapi kalimat (7) adalah ....
A. dengan demikian
B. oleh karena itu
C. di samping itu
D. untuk itu
E. jadi

Jawaban: B

153. Buah mangga memiliki kandungan vitamin C dan antioksidan sehingga dapat meningkatkan kekebalan tubuh dan membantu melawan infeksi virus. Penelitian terbaru memperlihatkan, mengonsumsi buah mangga bisa mencegah kanker dan menurunkan kadar kolesterol. 

Manakah DUA simpulan yang PALING DIDUKUNG oleh bacaan tersebut?
I. Harga buah mangga jadi mahal karena terdapat manfaat untuk kesehatan.
II. Jumlah buah mangga yang dikonsumsi berpengaruh terhadap khasiat buah mangga untuk mencegah kanker.
III. Banyak penderita kanker yang mencari buah mangga untuk pengobatan.
IX. Disarankan mengonsumsi buah mangga agar dapat terhindar dari risiko penyakit tertentu.
X. Banyak produk kesehatan yang memakai bahan dasar ekstrak buah mangga. 

Jawaban: III dan IX

154. Kebiasaan tidur dengan posisi tengkurap dinilai buruk untuk kesehatan karena memberikan tekanan terhadap bagian punggung dan leher yang berakibat nyeri. Selain itu, tidur tengkurap pun dapat menyebabkan kesemutan. Walau begitu, tidur tengkurap dapat membuka saluran pernapasan sehingga mengurangi dengkuran.

Berdasarkan bacaan tadi, manakah pernyataan di bawah ini yang PASTI BENAR?
A. Orang yang mengalami kesemutan tidur dengan posisi tengkurap.
B. Kesemutan karena tidur tengkurap akan menyebabkan tidur menjadi tidak nyenyak.
C. Orang yang gangguan pernapasan dapat disembuhkan dengan tidur tengkurap
D. Nyeri punggung dan leher dialami oleh orang yang mendengkur.
E. Dengkuran sebagian orang yang tidur tengkurap bisa berkurang.

Jawaban: E

155. Negara A adalah negara kepulauan dengan biaya logistik tinggi. Ahli logistik menjelaskan hal ini dikarenakan tidak meratanya infrastruktur di negara tersebut sehingga tidak semua wilayah bisa dijangkau dengan mudah.

Manakah pernyataan-pernyataan berikut yang MEMPERLEMAH dan TIDAK MEMPERLEMAH pendapat ahli logistik di atas?

Pernyataan:
I. Pemerintah membangun banyak infrastruktur baru guna memastikan pemerataan pembangunan.
II. Pemerintah menerapkan kebijakan insentif pajak bagi industri logistik untuk mengurangi beban biaya
III. Masyarakat memilih menggunakan jasa logistik saat berada pada pulau yang sama.
IV. Perusahaan logistik memastikan seluruh wilayah mampu dijangkau dengan mudah.
V. Konsumen memilih perusahaan logistik yang memberikan layanan jangkauan pengiriman yang luas.

Jawaban:
I. (memperlemah)
II. (tidak memperlemah)
III. (tidak memperlemah)
IV. (memperlemah)
V. (tidak memperlemah

156. Jika sudah siap panen, jeruk akan terasa empuk jika ditekan, kulitnya berwarna kuning langsat, dan apabila tergores, keluar aroma wangi. Inilah yang membuat nilai jual jeruk semakin tinggi. Apabila buah jeruk terasa keras saat ditekan, berwarna hijau, dan tidak mengeluarkan aroma, manakah kesimpulan yang BENAR?
A. Buah jeruk belum bisa dipanen.
B. Buah jeruk harganya murah.
C. Buah jeruk harganya mahal.
D. Buah jeruk tidak dapat dijual.
E. Buah jeruk rasanya masam.

Jawaban: A

157. Sekarang ini penggunaan kendaraan yang ramah lingkungan banyak dilakukan oleh masyarakat. Dengan pemanfaatan ini, konsumsi bahan bakar minyak di masyarakat mampu menghasilkan efisiensi sebesar 20%-30% setiap tahunnya.

Berdasarkan paragraf di atas, manakah yang PALING MUNGKIN menjadi asumsi yang mendasari argumen tersebut?
A. Masyarakat menjadi lebih hemat saat memakai kendaraan ramah lingkungan.
B. Kendaraan ramah lingkungan menjadi pilihan sebagian besar masyarakat.
C. Kesadaran masyarakat dalam memanfaatkan teknologi yang ramah lingkungan semakin meningkat.
D. Banyak produsen kendaraan yang memakai teknologi ramah lingkungan pada kendaraan produksinya.
E. Hemat energi menjadi program yang gencar dikampanyekan oleh pemerintah negara Y.

Jawaban: C

158. ..., 5, 7, 9, 11, 13.
Angka yang paling sesuai untuk melengkapi deret tersebut adalah ....
A. 0
B. 1
C. 2
D. 3
E. 4

Jawaban: D

159. Hadiah liburan ke luar negeri akan diberikan oleh perusahaan Y apabila pekerjanya mendapat penilaian kinerja sangat baik dua tahun berturut-turut dan dinominasikan oleh rekan kerja sekaligus atasannya. Pekerja X disukai oleh rekan kerjanya dan mendapatkan penilaian kinerja sangat baik tahun kemarin. Kesimpulan berdasarkan informasi tersebut yaitu pekerja X berhak memperoleh hadiah wisata ke luar negeri dari perusahaan Y.

Manakah pernyataan di bawah ini yang menggambarkan kualitas kesimpulan tersebut?
A. Kesimpulan tersebut pasti benar.
B. Kesimpulan tersebut mungkin benar.
C. Kesimpulan tersebut pasti salah.
D. Kesimpulan tidak relevan dengan informasi yang diberikan.
E. Kesimpulan tidak bisa dinilai karena informasi tidak cukup.

Jawaban: C

160. Dua dus minuman dibagikan untuk empat kelompok kerja dengan 
ketentuan berikut ini:
Kelompok B memperoleh 2 kali lebih banyak daripada kelompok C, Kelompok A memperoleh ½ bagian keseluruhan. Kelompok C dan D mendapatkan sama rata dari minuman yang tersisa. Setiap dus terdapat 24 kotak minuman ringan. Informasi yang bisa digunakan untuk mengetahui jumlah kotak minuman yang diterima oleh kelompok A adalah ....
A. 2 dan 4
B. 1 dan 3
C. 2 dan 3
D. 1 dan 2
E. 3 dan 4

Jawaban: A (2 dan 4)

Halaman ini akan selalu di update. Soal-soal UTBK tersebut cocok untuk dijadikan sebagai sarana latihan persiapan menghadapi ujian tersebut. Mudah-mudahan yang akan mengikuti seleksi tersebut diberikan kelulusan.

Referensi
https://www.detik.com/sulsel/berita/d-6730811/100-contoh-soal-utbk-snbt-2023-lengkap-kunci-jawaban-dan-pembahasan
https://www.brainacademy.id/blog/contoh-soal-tes-skolastik-utbk
https://www.sonora.id/read/423878071/80-contoh-soal-utbk-snbt-2023-lengkap-dengan-kunci-jawabannya
https://www.detik.com/edu/seleksi-masuk-pt/d-6705915/50-contoh-soal-utbk-snbt-2023-kunci-jawaban-dan-pembahasannya
LihatTutupKomentar
close